Breast Augmentation, Mastopexy Flashcards

1
Q

A 24-year-old woman comes to the office to discuss augmentation mammaplasty. She is interested in subglandular implant placement and would like to discuss the risks of augmentation. Which of the following risks is more likely with smooth round silicone implants compared with textured anatomic silicone implants?

A) Anaplastic large cell lymphoma
B) Capsular contracture
C) Double capsule
D) Late seroma
E) Malrotation

A

The correct response is Option B.

Capsular contracture is more common in smooth round silicone implants than in textured implants. It is believed that the texturing of the implant is protective against significant capsule formation.

On the other hand, there are several increased risks associated with textured anatomic implants. These include increased risks of late seroma and breast implant-associated anaplastic large cell lymphoma (BIA-ALCL), although this is very rare. Double capsule is a complication more recently noted with the introduction of textured anatomic implants. Malrotation can only be seen in an anatomic textured implant, because smooth round implants are symmetric in shape. In addition, it can be difficult to differentiate between anatomic shaped and smooth round implants, with several studies showing their similar cosmetic outcomes.

2018

How well did you know this?
1
Not at all
2
3
4
5
Perfectly
2
Q

A 23-year-old woman undergoes augmentation mammaplasty with round, smooth silicone implants placed in the dual-plane position. Postoperatively, unilateral erythema and warmth are noted, and they slowly resolve over 10 days of oral antibiotic treatment. The patient asks what this might mean for future satisfaction with the outcomes. Which of the following is the most likely sequela of this patient’s clinical course?

A) Breast gland ptosis
B) Capsular contracture
C) Double-bubble appearance
D) Implant rupture
E) Nipple numbness

A

The correct response is Option B.

One of the most often mentioned potential risk factors for capsular contracture is biofilm, and this may be related to bacterial contamination. History of infection is unlikely to impact nipple sensation, implant rupture, true breast gland ptosis, or effacement of the inframammary fold with downward descent of the implant.

2018

How well did you know this?
1
Not at all
2
3
4
5
Perfectly
3
Q

In women with breast ptosis, which of the following is an advantage of performing combined one-stage augmentation mammaplasty with mastopexy compared with mastopexy alone?

A) Better predictability of cosmetic outcome
B) Decreased complication rates
C) Decreased revision rates
D) Improved upper pole projection
E) Lower operative costs

A

The correct response is Option D.

Early reports have raised concerns about the safety of combined augmentation mammaplasty with mastopexy surgeries. However, in patients who wish to correct their breast ptosis these two procedures are often combined to a one-stage surgery and can show favorable outcomes. Nevertheless, plastic surgeons advocate that these cases should only be performed by experienced physicians.

Reasons are that the overall aesthetic results are harder to predict in one-stage augmentation/mastopexy procedures compared to mastopexy alone or even the two-stage augmentation mammaplasty followed by mastopexy. Both complication and revision rates are highest in the one-stage approach that combines augmentation mammaplasty with mastopexy. Longer operative time and the need for implants naturally increase operative costs.

The advantage of the simultaneous insertion of implants is the improved superior pole projection that cannot be achieved by mastopexy alone.

2018

How well did you know this?
1
Not at all
2
3
4
5
Perfectly
4
Q

In grade II ptosis of the breast, which of the following best describes the position of the nipple?

A) At the apex of the breast mound
B) At the lowest contour of the breast
C) At the transposed inframammary fold
D) Between the inframammary fold and the lowest contour of the breast
E) On the posterior aspect of the breast as it rests on the chest wall

A

The correct response is Option D.

The classic Regnault definition of breast ptosis classifications are as follows:

Grade I: Nipple at the level of the inframammary fold
Grade II: Nipple between the level of the inframammary fold and the lowest contour of the breast
Grade III: Nipple at the lowest contour of the breast

2018

How well did you know this?
1
Not at all
2
3
4
5
Perfectly
5
Q

A 35-year-old woman is evaluated for long-term follow-up 9 years after undergoing bilateral augmentation mammaplasty for hypomastia by another surgeon. The mammaplasty was performed with 450-mL smooth, round, silicone subglandular implants. The patient recently found out she is BRCA2 positive and underwent MRI of the breasts as part of a surveillance study. The MRI showed a “linguine sign” in the right breast. Which of the following findings on physical examination is most consistent with the diagnosis associated with the “linguine sign”?

A) The right breast has more rippling than the left breast
B) The right breast is not significantly different from the left breast
C) The right breast is significantly larger than the left breast
D) The right breast is significantly smaller than the left breast

A

The correct response is Option B.

The right breast is not significantly different from the left breast. The linguine sign describes multiple low-signal curvilinear lines on MRI that correlated to the collapsed implant shell. It is an indication of intracapsular rupture. Physical examination alone is not specific or sensitive enough to diagnose all cases of intracapsular rupture. Ultrasound and/or MRI is recommended. The physical examination finding of one breast that is smaller, firmer, and higher than the other is indicative of capsular contracture. MRI is not a sensitive predictor of capsular contracture. A right breast that is significantly larger than the left breast would indicate a late seroma and a workup for breast implant-associated anaplastic large-cell lymphoma would be indicated. A right breast that is significantly smaller than the left breast would be indicative of a ruptured saline implant. Increased rippling is not expected with an intracapsular rupture.

2018

How well did you know this?
1
Not at all
2
3
4
5
Perfectly
6
Q

A 40-year-old woman who underwent a subglandular augmentation mammaplasty with smooth round silicone breast implants 5 years ago returns to the office for evaluation of an increasingly firm left breast. Surgical revision of the left breast is planned. Which of the following measures is most likely to decrease the recurrence of the symptoms?

A) Conversion to a new plane or pocket
B) Performing a total capsulectomy
C) Using botulinum toxin type A in and around the implant pocket
D) Using fat grafting in and around the implant pocket

A

The correct response is Option A.

Site change and implant exchange are the only factors that have consistently been shown to decrease recurrence of capsular contracture, although other factors including use of a textured implant and fat grafting used are in augmentation mammoplasty revision. Botulinum toxin type A has been described for prevention of capsular contracture however; no consensus that these treatments decrease recurrence of capsular contracture exists.

Furthermore, there are no data to support performing total versus partial capsulectomy, or even the superiority of capsulectomy over capsulotomy.

2018

How well did you know this?
1
Not at all
2
3
4
5
Perfectly
7
Q

A patient comes to the office 6 months after undergoing bilateral vertical mastopexy because she is dissatisfied with her postoperative appearance. Height is 5 ft 5 in (165 cm). Physical examination shows the distance from nipple to sternal notch is 16 cm bilaterally, and the distance from nipple to inframammary fold is 14 cm bilaterally. Which of the following is the most appropriate next step in management?

A) Conversion to free nipple grafts
B) Placement of a dual-plane breast implant
C) Placement of a subglandular breast implant
D) Resection of excess skin at the level of the inframammary fold
E) Reassurance, massage, and observation

A

The correct response is Option D.

This case illustrates superior nipple malposition. The distance from nipple to inframammary (IMF) fold of 14 cm is much too long. The correct answer is to resect the lower pole skin at the IMF in order to move the nipple down. This would create a “T” scar and improve nipple position. Vertical mastopexies and reduction mammaplasties have a learning curve and much of this is predicting the nipple position postoperatively. The nipple should be designed lower on the breast than is done during marking a Wise pattern. At 6 months, it is unlikely the nipple position will change dramatically, so observation is not recommended. Addition of an implant will not help the nipple position. Conversion to free nipple grafts, while possible, will not lead to an aesthetic scar pattern.

2018

How well did you know this?
1
Not at all
2
3
4
5
Perfectly
8
Q

A 23-year-old woman with micromastia and bilaterally tuberous breasts comes to the office to discuss augmentation mammaplasty and improving the overall appearance of her breasts. Physical examination shows bilateral mildly ptotic breasts with glandular tissue herniating through the nipple-areola complex. The lower pole appears mildly deficient in the lower medial and lateral quadrants, and the distance from the nipple to the inframammary crease is 5.5 cm on stretch. Which of the following is the most appropriate operative approach for this patient?

A) Implant placement with circumareolar mastopexy
B) Implant placement without mastopexy
C) Implant placement with vertical mastopexy
D) Implant placement with Wise-pattern mastopexy
E) Two-stage reconstruction with tissue expander, followed by placement of a permanent implant

A

The correct response is Option A.

The tuberous breast deformity was first described by Aston and Rees in 1976. While most of the surgical approaches listed, with the exception of implants together with Wise-pattern mastopexy, have been described for the spectrum of tuberous breast deformities, the key is to select the right procedure for the right patient. In this case, a mild form of the deformity is described. Implant placement alone, even with parenchymal scoring and lowering of the inframammary crease, is unlikely to correct the deformity of the nipple-areola complex. In cases of severe ptosis, vertical mastopexy may be used but would be unnecessary in this patient with mild ptosis. In severely deficient cases, a two-stage approach with tissue expansion may be necessary, but it would be over-operating in this mildly deficient patient. Recently, fat grafting has also been advocated for this procedure.

In the case described, which is a common presentation, a periareolar approach is typically used to place the implant in a dual-plane configuration. Subglandular placement is also described. The inframammary crease is commonly adjusted downward. Radial scoring of the parenchyma and a circumareolar mastopexy are typically performed.

In the recent review by Kolker and Collins, 92% of tuberous breast patients had a one-stage procedure. Ninety-six percent of these were treated with implant placement and circumareolar mastopexy, combined with inframammary crease adjustment and radial scoring of the parenchyma.

2018

How well did you know this?
1
Not at all
2
3
4
5
Perfectly
9
Q

A 35-year-old woman is evaluated because of swelling of the right breast 3 years after undergoing augmentation mammaplasty. The implant type is unknown. Ultrasonography shows a seroma, and a fine-needle aspiration is performed. Which of the following immunohistochemical stains of the aspirate is most appropriate?

A) CCD79a
B) CD30
C) CK20
D) E-cadherin
E) p63

A

The correct response is Option B.

Patients who present with a late seroma should be evaluated for possible breast implant-associated anaplastic large cell lymphoma (BI-ALCL). A late seroma is usually accepted as occurring 1 year following surgery; however, there are cases of BI-ALCL seromas that have presented as early as 4 months.

The first step in evaluating BI-ALCL is ultrasonography, followed by fine-needle aspiration if indicated. The fluid requires evaluation beyond routine cell cytology. Immunohistochemistry test for CD30 was the most commonly positive marker for BI-ALCL. Immunohistochemistry stains specific antigens in cells by binding to this antigen in an antibody/antigen reaction. The specific stain can then be seen under light microscopy. The CD30 antibody labels anaplastic large cell lymphoma cells. CD30 is a transmembrane cytokine receptor belonging to the tumor necrosis factor receptor family.

CK20 and CCD79a were negative for tested BI-ALCL specimens.

P63 stains myoepithelial cells and is used to rule out invasive breast tumors.

E-cadherin helps distinguish ductal from lobular carcinoma.

2018

How well did you know this?
1
Not at all
2
3
4
5
Perfectly
10
Q

A 58-year-old woman undergoes removal of round 280-cc silicone gel implants she has had for over 30 years. New silicone gel implants measuring 10 cm in width by 12 cm in height with a 5-cm projection are placed. Compared with her original gel implants, the new implants are more likely to have a higher rate of which of the following complications?

A) Contracture
B) Infection
C) Rippling
D) Rotation
E) Rupture

A

The correct response is Option D.

Breast implant technology has evolved greatly since implants were introduced in the 1960s. Increased cross-linking of silicone polymers (polydimethylsiloxanes) results in a more stable, cohesive form and closer shell-gel interactions. Advantages of these more “form-stable” implants include lower rates of rippling and rupture. They allow for the creation of shaped implants that offer clear advantages for certain patients, such as those seeking a natural upper pole shape transition, and those with wider or taller breast shapes.

The biggest drawback of shaped implants is the need to place them in a precise surgical pocket lest they rotate, causing deformity and potentially requiring reoperation. As long as surgeons follow sound surgical principles of dissecting an appropriate pocket limited to the approximate width of the implant, malrotation rates are low, typically in the 1.5% range. In one study, half of patients with implant rotation improved with manual repositioning and taping for 3 to 6 weeks, while the other half required reoperation.

Infection rates do not vary among implant types.

Shaped implants have textured shells, which have been shown to have lower rates of capsule contracture, particularly in the subglandular position.

Implant rupture rates are also lower in new generation implants, in the 0.7% per year range.

Visible rippling rates are more common in thinner consistency implants, such as saline and older silicone devices.

2017

How well did you know this?
1
Not at all
2
3
4
5
Perfectly
11
Q

Breast implant–associated anaplastic large cell lymphoma is most closely associated with which of the following implant characteristics?

A) High-profile dimensions
B) Saline filling
C) Silicone gel filling
D) Smooth shell
E) Textured shell

A

The correct response is Option E.

The overwhelming majority of reported cases of breast implant–associated anaplastic large cell lymphoma (BIA-ALCL) have been associated with textured surface implants.

Anaplastic large cell lymphoma in association with breast implants is a rare occurrence; however, when it does appear, the course is usually less aggressive with a better prognosis than when it is unrelated to breast implants. Recent studies suggest that the breast implant shell causes a chronic T-cell stimulation. This reaction is thought to be caused by an interaction of textured surface characteristics and associated biofilm.

BIA-ALCL associated with smooth shell implants has occurred; however, it is disproportionately rare.

ALCL is seen with both silicone- and saline-filled implants. These numbers are highly influenced by the specific popularity of each implant. Specific implant dimensions, be it projection or width, are not uniquely associated with ALCL.

2017

How well did you know this?
1
Not at all
2
3
4
5
Perfectly
12
Q

Which of the following is the most common cause of litigation in cosmetic breast surgery?

A) Assault
B) Failure to diagnose or treat an injury related to the procedure
C) Lack of informed consent
D) Negligence
E) Retained surgical instrument

A

The correct response is Option D.

Plastic surgery faces one of the highest proportions of malpractice claims compared with other medical specialties. A number of studies have revealed that breast-related surgeries account for 37% of overall claims against plastic surgeons. The most common cause of action is negligence, related either to lack of appropriate knowledge or skill or to failing to meet the standard of care. The second most common cause of action is lack of informed consent. Lack of informed consent results from the failure of the physician to thoroughly discuss the risks associated with surgery and the options of alternative therapies. Other causes of action include failure to diagnose or treat injury related to the procedure, retained surgical materials, assault, and distortion of physician’s credentials.

2017

How well did you know this?
1
Not at all
2
3
4
5
Perfectly
13
Q

An otherwise healthy 28-year-old woman comes to the physician requesting removal of bilateral axillary masses. She states that the masses fluctuate in size and tenderness with her menstrual cycle. She reports that the masses have not had discharge or drainage. Physical examination shows smooth, spongy masses in both axillae. A photograph is shown. Which of the following is the most appropriate next step in management of this patient?

A) Excise the bilateral axillary masses and skin
B) Obtain bilateral mammograms of the axillary masses
C) Order an MRI of the chest
D) Perform a core biopsy of both axillary masses
E) Perform liposuction

A

The correct response is Option A.

This patient presents with ectopic breast tissue. In utero, the milk line (galactic band) forms at 5 weeks of gestation. This bilateral structure courses from the axillae to the groin, and normal breasts form in the prepectoral region after there has been regression of the rest of the galactic band. When there is failure of this regression, breast tissue remains in locations outside of the normal breast. The most common location for ectopic breast tissue is in the axillae, although it can be found anywhere along the milk line from the axillae to the groin. Ectopic breast tissue outside of the milk line has been described and is termed aberrant breast tissue.

The tissue found in these ectopic locations is breast tissue with the same characteristics and propensity for disease as normally located breast tissue, and breast cancer has been described in these tissues. In the absence of pathologic findings such as a mass, pain, and skin changes that are associated with breast cancer, there is no strong oncologic indication for excision. If there are findings concerning for a neoplasm, then work-up should be initiated and might include further imaging, core biopsy, and surgery. However, most cases present without pathologic findings and are excised to achieve a more reasonable appearance for the patient, the ability to don clothing more comfortably, and for the obvious social advantages.

In this case, the patient is young, has no complaints, and has no physical findings to suggest a neoplasm. Excision should be offered.

Obtaining bilateral mammograms is incorrect because there is no indication for imaging in this patient based on her age, history, and physical examination. In addition, mammograms of axillary breast tissue are technically unfeasible.

Performing a core biopsy is incorrect as there is no concern for malignancy in this case. In the case of a mass noted within the ectopic axillary breast tissue, then an oncologic workup should be initiated which might include a core biopsy.

An MRI of the chest is incorrect because there is no indication for imaging in this patient based on her age, history, and physical examination.

Reassuring the patient with no further action is not the most appropriate management, as it will not address the patient’s concerns and desires. In the patient who does not request excision or is not an appropriate surgical candidate, then reassurance and surveillance are appropriate.

As this is a young female with axillary breast tissue, liposuction will not improve the excess breast tissue or skin.

2017

How well did you know this?
1
Not at all
2
3
4
5
Perfectly
14
Q

In embryologic breast development, which of the following best describes the formation of the mammary ridge?

A) Starts at the fifth or sixth week of fetal development, when buds of mesoderm grow into the overlying ectodermal skin layer
B) Starts at the fifth or sixth week of fetal development, when outgrowths from the ectodermal skin layer penetrate into the mesoderm
C) Starts at the seventh or eighth week of fetal development, when buds of mesoderm grow into the overlying ectodermal skin layer
D) Starts at the seventh or eighth week of fetal development, when outgrowths from the ectodermal skin layer penetrate into the mesoderm
E) Starts at the third or fourth week of fetal development, when buds of mesoderm grow into the overlying ectodermal skin layer

A

The correct response is Option B.

Muntan, et al. described breast development as starting at the fifth or sixth week of development, when outgrowths from the ectodermal skin layer penetrate into the underlying mesoderm, forming the mammary ridge or milk line. The ectodermal thickenings along the mammary line regress between gestational months 2 and 4, except for two of them in the region of the third and fourth ribs. The ectoderm keeps on extending into the underlying mesoderm at the fifth month, and a branching network forms what will eventually become the lactiferous system. The supportive connective and adipose tissue of the breast develops from the surrounding mesenchyme.

2017

How well did you know this?
1
Not at all
2
3
4
5
Perfectly
15
Q

A 36-year-old woman, gravida 3, para 3, comes to the physician because she desires larger breasts. She has breast-fed three children. Physical examination shows grade 3 ptosis and loss of superior pole volume. The distance from nipple to sternal notch is 26 cm. Result of upper pole pinch test is 1.5 cm. A dual-plane augmentation/mastopexy is planned. Which of the following is the strongest indicator for subpectoral placement of the implant in this patient?

A) Concurrent mastopexy
B) Grade 3 ptosis
C) Loss of superior pole volume
D) Nipple to sternal notch distance of 26 cm
E) Pinch test result of 1.5 cm

A

The correct response is Option E.

A dual-plane approach is subpectoral in the superior pole and subglandular in the inferior pole. This is to afford more subcutaneous coverage in the superior pole. Tebbetts recommends pinching the skin and subcutaneous tissues of the superior pole for a “pinch test.” For thickness less than 2 cm, he recommends a dual-plane placement for adequate soft tissue coverage. This is not affected by the grade of ptosis, need for mastopexy, history of loss of superior pole volume, or nipple to notch distance.

2017

How well did you know this?
1
Not at all
2
3
4
5
Perfectly
16
Q

Which of the following is most commonly associated with decreased incidence of capsular contracture?

A) Formation of biofilm
B) Placement of textured silicone device
C) Subglandular placement of the implant
D) Use of a periareolar incision
E) Use of a postoperative surgical brassiere

A

The correct response is Option B.

The rest of the options have been shown to increase the incidence of capsular contracture. Textured silicone implants, inframammary incisions, and submuscular implant placement have been shown to decrease the incidence of capsular contracture. The use of a surgical brassiere postoperatively has not been shown to decrease incidence of capsular contracture as well.

2017

How well did you know this?
1
Not at all
2
3
4
5
Perfectly
17
Q

In augmentation mammaplasty, which of the following is the ideal upper pole to lower pole anatomic ratio?

A) 25:75
B) 35:65
C) 45:55
D) 50:50
E) 55:45

A

The correct response is Option C.

Studies have demonstrated the ideal anatomical characteristics of the breast to include: an upward pointing nipple, a straight or mildly concave upper pole slope, smooth lower pole convexity and fuller lower pole compared to upper pole. Breasts with an upper pole-to-lower pole ratio of 45:55 were identified as defining the ideal breast. The ratio was defined ideal by respondents including women, men, plastic surgeons, and individuals of ethnic diversity.

2017

How well did you know this?
1
Not at all
2
3
4
5
Perfectly
18
Q

A 58-year-old woman with moderate ptosis is evaluated for mastopexy. According to Regnault classification, which of the following best describes the location of the nipple-areola complex in type II breast ptosis?

A) 1 to 3 cm inferior to the inframammary fold
B) 4 cm inferior to the inframammary fold
C) 6 cm inferior to the inframammary fold
D) At or 1 cm inferior to the inframammary fold
E) Superior to the inframammary fold

A

The correct response is Option A.

Regnault classification of breast ptosis, based on the position of the nipple-areola complex (NAC) relative to the inframammary fold (IMF):

The type of mastopexy performed will depend on the degree of breast ptosis. Breast ptosis is graded using Regnault classification. Type I can be treated with a crescent mastopexy, when the degree of nipple-areola complex elevation does not exceed 1 cm. Type I or II ptosis can be treated with a periareolar mastopexy, when the distance of nipple-areola complex elevation ranges from 1 to 2 cm. Type II and III ptosis is amenable to the inverted-T technique, where the horizontal incision will reduce the distance from the nipple-areola complex to the inframammary fold, while the vertical incision will reduce the base diameter.

How well did you know this?
1
Not at all
2
3
4
5
Perfectly
19
Q

A 48-year-old woman, gravida 3, para 3, who wears a size 36B bra comes to the physician for evaluation of breast ptosis. BMI is 24 kg/m2. Physical examination shows the distance from sternal notch to nipple is 28 cm, and there is grade 2 breast ptosis with skin laxity. A combined augmentation/mastopexy is planned. Which of the following is the biggest risk of combining the procedures rather than staging them?

A) Hematoma
B) Need for revision procedure
C) Nipple-areola complex necrosis
D) Seroma
E) Transection of lateral intercostal nerves

A

The correct response is Option B.

Combining an augmentation with a mastopexy has long been considered risky because the surgeon is addressing two opposing forces during the same operation: the ptosis and volume, for which the placement of additional weight may exacerbate ptosis. Studies have shown, however, that the two operations can safely be combined. During the planning, particularly for severe ptosis, the surgeon must be careful not to overresect skin that will be critical for closure over an implant.

Compared with staged procedures, mastopexy-augmentation has a higher rate of need for revision procedures. Patients should be counseled about the potential need for revisions.

Seroma and hematoma are not increased when combining the procedures, and nipple-areola complex necrosis is a function of pedicle size and patient-specific factors such as obesity and tobacco use, rather than the combination of procedures. Similarly, transection of intercostal nerves is associated more closely with pedicle type than with combining procedures.

2017

How well did you know this?
1
Not at all
2
3
4
5
Perfectly
20
Q

Which of the following cell types is most associated with the chronic inflammation that leads to breast implant–associated anaplastic large cell lymphoma?

A) B-cells
B) Monocytes
C) Neutrophils
D) Red blood cells
E) T-cells

A

The correct response is Option E.

Evidence suggests that chronic inflammation is the stimulus responsible for the development of breast implant–associated anaplastic large cell lymphoma (ALCL) and T-cells are the predominant cell type responding to this antigenic stimulus. B-cells have been implicated in orthopedic implant lymphomas. The other cell types are involved in inflammation, but they are not associated with breast implant-associated ALCL.

2017

How well did you know this?
1
Not at all
2
3
4
5
Perfectly
21
Q

A 38-year-old woman reports decreased areola sensitivity after undergoing mastopexy. Intraoperative injury to which of the following nerves is the most likely cause of this patient’s reduced sensitivity?

A) Intercostobrachial nerve
B) Lateral cutaneous branch of the fourth intercostal nerve
C) Lateral cutaneous branch of the sixth intercostal nerve
D) Medial cutaneous branch of the fifth intercostal nerve
E) Medial cutaneous branch of the third intercostal nerve

A

The correct response is Option B.

The lateral cutaneous branch of the fourth intercostal nerve is most commonly responsible for nipple and areola sensitivity. The other intercostal nerve branches listed do contribute to breast sensitivity but are less often thought to be the primary innervation to the nipple and areola. The intercostobrachial nerve supplies innervation to the upper medial arm.

2017

How well did you know this?
1
Not at all
2
3
4
5
Perfectly
22
Q

A 47-year-old woman, gravida 2, para 2, who has grade III breast ptosis is evaluated for mastopexy. Attenuation of which of the following structures is the most likely cause of the ptosis?

A) Breast acini
B) Cooper ligaments
C) Lactiferous ducts
D) Scarpa’s fascia
E) Subdermal plexus

A

The correct response is Option B.

Breast ptosis is a complex interaction of events, informed by breast size, gravity, aging, lactation, and parity. It occurs through a combination of atrophy of the breast tissue, loss of elasticity of the skin envelope, and attenuation of Cooper ligaments.

While the breast is surrounded by fascia, the continuation of Scarpa’s fascia forms the posterior capsule of the breast.

The lactiferous ducts and breast acini do not contribute significantly to ptosis.

Subdermal plexus provides vascularity rather than support to the breast.

2017

How well did you know this?
1
Not at all
2
3
4
5
Perfectly
23
Q

A 28-year-old woman comes to discuss primary augmentation mammaplasty options and is deciding between form-stable shaped implants and less cohesive round silicone gel implants. She inquires about the benefits of each type of implant. Compared with smooth round silicone gel implants, highly cohesive form-stable gel implants have a decreased incidence of which of the following?

A) Capsular contracture
B) Implant malposition
C) Infection
D) Seroma

A

The correct response is Option A.

Form-stable silicone gel implants are fifth-generation, shaped, and textured implants that have additional cross-linking between molecules. They are purported to have several advantages over other round saline and silicone gel implants because they retain their shape and decrease the incidence of folding and rippling. This has translated into significantly lower capsular contracture rates.

However, they do have some disadvantages. Because they are shaped and maintaining orientation is critical, they have a higher incidence of malposition. They are also more prone to seroma formation, which may be associated with their textured surface.

Infection and resorption rates remain similar.

2016

How well did you know this?
1
Not at all
2
3
4
5
Perfectly
24
Q

A 28-year-old woman comes to the office to discuss augmentation mammaplasty. She is interested in silicone implants, specifically highly cohesive gel shaped implants. Which of the following is the most likely result of increasing the cross-linking in these implants?

A) Decreased risk of gel fracture
B) Decreased risk of shell delamination
C) Improved form stability
D) Increased risk of folds
E) Softer implants

A

The correct response is Option C.

Increasing the cross-linking in a highly cohesive gel shaped silicone implant improves form stability. This allows for the creation of shaped implant designs that persist despite position or external forces on the implant.

The current, fifth-generation silicone breast implants derive their cohesiveness from the cross-linking of the silicone. Increasing the amount of cross-linking leads to an increase in cohesiveness and a firmer implant. This may lead to less rippling and folding because of resistance to collapse; however, recent MRI studies have shown folds and distortions are still possible. Increasing cohesiveness, however, does have some disadvantages with potential risks for gel fracture and delamination of the implant shell.

2016

How well did you know this?
1
Not at all
2
3
4
5
Perfectly
25
Q

A 13-year-old girl is evaluated for breast asymmetry. Examination shows total absence of the left mammary gland tissue, with normal areola and nipple. Pectoral muscles are normal. No hand, facial, or other body abnormalities are noted. Which of the following is the most likely diagnosis?

A) Amastia
B) Amazia
C) Athelia
D) Ectodermal dysplasia
E) Poland sequence

A

The correct response is Option B.

There are a number of uncommon aplastic deformities of the breast. These include: total absence of the breast and nipple (amastia), absence of the nipple (athelia), and absence of the mammary gland (amazia), as described in this case. These anomalies may occur in isolation, or may be associated with various syndromes, such as Poland syndrome, where the absence of the breast is associated with absence of the pectoralis major muscle, rib cage and ipsilateral upper limb deformities. Ectodermal dysplasias can affect the breast, but two or more abnormalities of ectodermal structures – hair, teeth, nails, sweat glands, craniofacial structures – would be required to consider the diagnosis.

2016

How well did you know this?
1
Not at all
2
3
4
5
Perfectly
26
Q

A 48-year-old woman comes to the office because she is very unhappy with the appearance of her breasts following a bilateral mastopexy performed 1 year ago. Height is 5 ft 7 in (170 cm). BMI is 26 kg/m2. Which of the following findings on physical examination would be most difficult to correct?

A) Asymmetrical breast size
B) Dog ear of the inferior vertical scar
C) Nipple to inframammary crease distance of 16 cm
D) Nipple to sternal notch distance of 16 cm
E) Widened circumareolar scar

A

The correct response is Option D.

A sternal notch to nipple distance of 16 cm represents a high-riding nipple. Revisional surgery for correction of a high-riding nipple is complex, and it is difficult to achieve a favorable result because of the surgeon’s and patient’s desire to avoid a scar extending superior to the nipple areola. Further, the paucity of excess skin between the nipple and clavicle limits the reconstructive options.

Suggested strategies include direct reposition of the nipple-areola complex, expansion of the skin between the nipple and clavicle, and repositioning of the breast parenchyma and inframammary crease.

Breast size asymmetry can be improved with either liposuction or revision mastopexy/reduction. The operation is usually performed using the previous incisions.

A dog ear of the inferior vertical scar is easily revised with a small transverse scar within the inframammary crease. The majority of these early postoperative deformities will resolve without surgery.

Recurrence of ptosis or an elongation of the nipple to inframammary crease distance occurs with all mastopexy operations. When performing secondary mastopexy, this can be improved with shortening the vertical scar with wedge resection at the inframammary crease. Knowledge of the location of the previous nipple areola pedicle is helpful in minimizing vascular complications.

Widened circumareolar scars can be revised with excellent results. Utilizing a permanent suture around the areola helps control size of the areola and tension on the suture line.

2016

How well did you know this?
1
Not at all
2
3
4
5
Perfectly
27
Q

The mammary glands develop from which of the following embryologic structures?

A) Bilateral mesenchymal condensations
B) Ingrowths from the ectoderm
C) Ingrowths from the mesoderm
D) Proliferating masses of endoderm
E) Proliferating masses of mesenchyme

A

The correct response is Option B.

The breasts, or mammary glands, are modified sweat glands. They are ingrowths from the ectoderm that form the lactiferous ducts and alveoli. They begin as linear mammary ridges with 15 to 20 buds. During the seventh week in utero, these buds undergo apoptosis, leaving a single pair of solid buds—the primary mammary buds—at the fourth or fifth intercostal space.

Proliferating masses of mesenchyme are at the center of each limb bud. The mesoderm gives rise to organs, musculature, vasculature, and connective tissues. The endoderm becomes the epithelial lining of the alimentary tract. Bilateral mesenchymatous condensations develop into the sternum.

2016

How well did you know this?
1
Not at all
2
3
4
5
Perfectly
28
Q

A 22-year-old nulliparous woman is evaluated for improvement of breast shape and size. Examination shows bilateral hypoplastic breasts with constricted bases and herniation of breast parenchyma in the areolae. Tuberous breast deformity is diagnosed. Bilateral breast augmentation with smooth, round gel implants via periareolar incisions is planned. Which of the following maneuvers is most likely to decrease the risk for a “double-bubble” deformity?

A) Decreasing the areolar diameter
B) Lowering of the inframammary fold
C) Parenchymal scoring
D) Periareolar incision
E) Subpectoral placement of the implant

A

The correct response is Option C.

Common hallmarks of tuberous breast deformity include varying degrees of hypoplastic breast parenchyma, deficiencies of the inferior pole, herniation of the parenchyma in the areola, enlarged areolae, superior placement of the inframammary fold, and asymmetry. Surgical goals are to achieve symmetry, sufficient volume (especially in the hypoplastic areas), lowering of the inframammary fold, reduction of areolar tissue herniation, and correction of any ptosis. A double-bubble deformity can occur when the inframammary fold is not sufficiently obliterated. The risk for this is increased with superiorly displaced inframammary folds, as in tuberous breasts. Parenchymal scoring would both release any constricting bands to allow the lower pole tissue to spread over the implant as well as release the superiorly displaced inframammary fold. While decreasing the areolar diameter and lowering of the inframammary fold are goals for breast improvement, neither will treat a double-bubble deformity. A periareolar incision is often advocated in repair of tuberous breasts because of the ability to reduce the areola; it alone, however, will not prevent a double-bubble deformity. Subpectoral placement of implants increases the risk for double-bubble deformity while subglandular placement of implants decreases the risk. Many advocate a dual-plane approach to capitalize on increased upper pole coverage combined with the benefits of a subglandular relationship in the inferior pole.

2016

How well did you know this?
1
Not at all
2
3
4
5
Perfectly
29
Q

A 35-year-old woman, gravida 2, para 2, seeks implant-based augmentation mammaplasty. She breastfed both her children. Which of the following is the most common complication of this procedure?

A) Early implant rupture
B) Hematoma
C) Infection
D) Lifetime need for reoperation
E) Seroma

A

The correct response is Option D.

Augmentation mammaplasty is known to have high rates of complications including reoperation. Infection, seroma, hematoma, and early implant rupture are rare in elective, cosmetic augmentation mammaplasty.

2016

How well did you know this?
1
Not at all
2
3
4
5
Perfectly
30
Q

An 18-year-old woman comes to the office for evaluation of her breasts. Photographs of the patient are shown. Which of the following statements most accurately describes the anatomy of this patient’s breasts?

A) The areola is normal size although the breast is small
B) The breast tissue is uniformly distributed throughout the breast pocket
C) The inframammary fold is elevated
D) The skin envelope has greater laxity than in a normal breast
E) The underlying musculature is underdeveloped

A

The correct response is Option C.

The tuberous breast deformity results in a protruding, oblong shape that resembles a tuberous root plant (Latin derivation tuber = to swell). The features noted in the tuberous breast deformity include a constricted breast base, decreased breast parenchyma, abnormal elevation of the inframammary fold, a decreased skin envelope, and herniation of the breast parenchyma through the central breast and into the areola. The areola is large and lacks firm underlying structure, thus allowing the breast tissue to protrude through this path of least resistance. The deformity is also often referred to as a tubular breast, constricted breast, doughnut breast, nipple breast, breast with narrow base, dome nipple, and snoopy dog breast.

The overall etiologic factors leading to the full expression of the constricted breast deformity are still largely unknown and likely involve a delicate balance of anatomic and endocrinologic forces. A constricting fibrous ring at the level of the areola periphery, representing probably a thickening of the superficial fascia coupled with the normally absent fascial layer in the NAC, has been proposed as a likely cause. The ring is composed of dense fibrous tissue made of large concentrations of collagen and elastic fibers arranged longitudinally. It is usually denser at the lower part of the breast and does not allow the developing breast parenchyma to expand during puberty. It has been suggested that a thickening of the superficial fascia combined with the absence of a superficial fascial layer under the NAC is the underlying anatomic/histopathologic cause of the deformity. The cause of the thickened fascia is unknown, although at least one study by Klinger, Caviggioli, et al. demonstrated altered collagen in both disposition and quantity. The same study excluded amyloid deposition as a component of the fibrosis.

The areola in the tuberous breast still contains the normal muscular structures that result in areolar changes with stimulation and temperature changes, although the tissue beneath the areola may be thinned.

2016

How well did you know this?
1
Not at all
2
3
4
5
Perfectly
31
Q

A 28-year-old woman is evaluated for micromastia. During consultation, she reports that her best friend underwent breast augmentation that was complicated by painful capsular contracture. Which of the following measures is most likely to prevent this complication in this patient?

A) Initiation of implant massage on postoperative day 5
B) Placement of a closed suction drain for prevention of postoperative hematoma
C) Use of a surgical support bra postoperatively for 2 weeks
D) Use of a subglandular, smooth, round implant via periareolar incision
E) Use of a subpectoral, textured implant via inframammary incision

A

The correct response is Option E.

Capsular contracture occurs when there is fibrosis of the peri-implant capsule. The severity is typically described by the Baker Grade classification.
Grade 1: the breast is soft and appears normal in size and shape
Grade 2: the breast is a little firm and appears normal
Grade 3: the breast is firm and appears abnormal
Grade 4: the breast is firm, appears abnormal, and is painful

Studies have shown a decreased relative risk for Baker grade 3-4 capsular contracture in primary breast augmentation associated with inframammary fold incision, textured implants, and subpectoral placement. The relative risk for capsular contracture was increased with periareolar or axillary incision, smooth implants, and subglandular placement. There is no evidence that wearing a support bra or implant massage will decrease the risk for capsular contracture. While hematoma is linked to capsular contracture, the presence of a drain does not prevent hematoma.

2016

How well did you know this?
1
Not at all
2
3
4
5
Perfectly
32
Q

A 47-year-old woman, gravida 3, para 3, is evaluated for improvement of breast appearance. She breast-fed all three of her children for 1 year each. Examination shows the distance from nipple to sternal notch is 27 cm bilaterally; decreased superior pole volume, and striae are also noted. There is Grade 3 ptosis bilaterally. The pinch of the superior pole soft tissue is 1 cm. Which of the following procedures is most likely to improve superior pole volume and breast shape in this patient?

A) Dual-plane implant augmentation
B) Mastopexy with dual-plane implant augmentation
C) Mastopexy with subglandular implant augmentation
D) Subglandular implant augmentation
E) Vertical mastopexy

A

The correct response is Option B.

Goals of improvement would be upper pole fullness and a coned, rounded breast, with raising the nipple. Because the superior pole thickness is less than 2 cm, a subglandular implant is not recommended. A dual-plane implant would not address the ptosis and would likely leave persistent ptosis. Vertical mastopexy alone would require some modification to address the excess vertical skin with some element of horizontal inferior excision. This would not address the lack of upper pole volume in the long term. The striae indicate poor tissue strength. Staged implant placement would have the fewest risks.

2016

How well did you know this?
1
Not at all
2
3
4
5
Perfectly
33
Q

A 55-year-old postmenopausal woman desires improvement in the appearance of her breasts. Change in which of the following levels of hormones is most likely responsible for postmenopausal involution of breast tissue?

A) Estrogen
B) Growth hormone
C) Oxytocin
D) Prolactin
E) Testosterone

A

The correct response is Option A.

Estrogen is the primary hormone in promoting the development of the breast epithelium and ductal tissue. Progesterone acts in combination with estrogen to regulate breast development. With the onset of menopause, there is a decrease in the secretion of estrogen and progesterone. As a result of the decrease in the circulating levels of these hormones, the breast undergoes regression and atrophy of the glandular elements.

Oxytocin and prolactin are hormones involved in the physiology of lactation. Growth hormone and testosterone may have an effect on breast tissue, but they are not primary factors in the physiology of the female breast.

2016

How well did you know this?
1
Not at all
2
3
4
5
Perfectly
34
Q

A 28-year-old woman, gravida 2, para 2, undergoes augmentation mammaplasty 1 year post partum. On postoperative day 3, the patient comes to the office because of impaired wound healing at the incision site. Physical examination shows white viscous discharge leaking from the edge of the wound consistent with galactorrhea. Which of the following is the most appropriate management?

A) Administration of bromocriptine
B) Administration of metoclopramide
C) Administration of trimethoprim-sulfamethoxazole
D) Application of negative pressure wound therapy
E) Debridement of the wound edges with wet-to-dry dressings

A

The correct response is Option A.

There are incidents of surgical procedures of the breast associated with galactorrhea leading to skin breakdown, nipple necrosis, and cellulitis. A dopamine agonist such as bromocriptine will cause decreased lactation in cases of galactorrhea/galactocele, thereby improving wound healing. Antibiotics such as sulfamethoxazole and trimethoprim (Bactrim) are generally not required, because the exudate is sterile. There is no need for debridement of the wound edges. Negative pressure wound therapy may increase lactation and galactorrhea, further impairing wound healing. Metoclopramide is a dopamine antagonist used for nausea and vomiting.

2016

How well did you know this?
1
Not at all
2
3
4
5
Perfectly
35
Q

A 25-year-old woman comes to the office because she is dissatisfied after undergoing breast augmentation mammaplasty for correction of tuberous breast deformities. Physical examination shows two parallel creases running transversely across the lower pole of each breast with inferior displacement of the implant. Which of the following best describes the position of the original inframammary fold in this patient?

A) Above the superior and inferior transverse creases
B) At the inferior transverse crease
C) At the superior transverse crease
D) Below the superior and inferior transverse creases

A

The correct response is Option C.

A double-bubble breast deformity following breast augmentation mammaplasty is represented by the development of two parallel, curvilinear transverse lines in the lower pole of the breast. The native inframammary fold is disrupted and represented by the superior transverse line. The lower transverse line represents the lower limit of implant pocket dissection or the final position of implant descent.

Predisposing anatomic factors for the development of a double-bubble deformity include tuberous breasts, constricted inframammary folds, or a short inframammary fold-to-nipple distance. Other factors that can increase the risk for the development of a double-bubble deformity include glandular ptosis, postpartum involution of the breasts, excessive implant size, and overdissection of the implant pocket. Correction of the double-bubble deformity may require conversion of the implant to a subglandular position, capsulorrhaphies, use of form-stable implants, or dermal grafts.

2016

How well did you know this?
1
Not at all
2
3
4
5
Perfectly
36
Q

A 24-year-old nulliparous woman comes to the office for augmentation mammaplasty. She currently wears a size 34B brassiere and wants her brassiere size to be increased to a D cup. She is a good candidate for subglandular placement of implants. Which of the following risks is decreased by the use of the textured silicone shell compared with the smooth silicone shell?

A) Capsular contracture
B) Hematoma
C) Prosthesis malposition
D) Rippling
E) Symmastia

A

The correct response is Option A.

Texturing of the implant surface has been shown to decrease the rate of capsular contracture when compared with smooth implants when the implants are placed in the subglandular position. The benefit of textured implants may not be present when the implants are placed in a submuscular pocket.

There is no difference in hematoma rates for textured versus smooth implants. Both symmastia and implant malposition are related to pocket dissection and not related to the type of implant placed. In the case of symmastia, the pockets have encroached upon the sternum and are close to each other or are touching. Implant malposition can be related to factors such as inadequate dissection of the pocket, or over-dissection of the pocket. Finally, some studies have demonstrated an increase in rippling with textured implant when compared with smooth implants. However, rippling may be more related to cohesiveness of the gel and fill volumes of the shell, because early reports of experience with the form-stable implant (Natrelle 410) seem to show decreased rates of rippling.

2015

37
Q

A 49-year-old woman is evaluated because of a traumatic laceration of the right lower eyelid and cheek. Physical examination shows difficulty with eyelid closure, voluntary squinting, and animation. Which of the following branches of the facial nerve is most likely injured?

A) Buccal
B) Cervical
C) Marginal mandibular
D) Temporal
E) Zygomatic

A

The correct response is Option E.

Anatomically, the orbicularis oculi muscle is divided into three segments: pretarsal, preseptal, and orbital. However, functionally, the orbicularis oculi muscle is divided into the medial inner canthal orbicularis and the extracanthal orbicularis. The medial inner canthal orbicularis is responsible for blinking, lower lid tone, and the pumping mechanism of the lacrimal system. Innervation to the inner canthal orbicularis is from the buccal branches of the facial nerve. The zygomatic branch of the facial nerve innervates the extracanthal orbicularis, which controls eyelid closure, voluntary squinting, and animation. The temporal, marginal mandibular, and cervical branches do not provide innervation to the orbicularis oculi muscle.

2015

38
Q

Which of the following is the most common complication associated with “donut” mastopexy?

A) Boxy breast shape
B) Increased distance from nipple to inframammary fold
C) Loss of nipple sensation
D) Nipple necrosis
E) Widening of the areola

A

The correct response is Option E.

A common complication of the “donut” (circumareolar) mastopexy is widening of the areola. This can be minimized by using a Gore-Tex suture placed using the “wagon-wheel” technique and limiting the amount of skin resected to a 2:1 ratio of outside diameter to areolar diameter.

Boxy breast shape is associated with Wise pattern mastopexy. Nipple necrosis is associated with combined augmentation and mastopexy. Increased distance from the nipple to the inframammary fold is associated with vertical mastopexies in which the height of the medial and lateral pillars is too tall. Loss of nipple sensitivity is unusual because there is no parenchymal resection.

2015

39
Q

A 27-year-old woman is evaluated because of pain 2 weeks after undergoing subglandular augmentation mammaplasty. She has no history of fever, chills, or drainage. Physical examination discloses a painful, tender cord in the inframammary region of the left breast. Which of the following is the most appropriate next step in management?

A) Administration of an antibiotic
B) Administration of an anticoagulant
C) Administration of an anti-inflammatory agent
D) Duplex ultrasonography
E) Removal of the implant

A

The correct response is Option C.

Mondor disease of the breast is a benign, self-limiting thrombophlebitis of the inframammary veins. Clinically, Mondor disease usually occurs 2 to 3 weeks postoperatively as a painful, tender cord within the superficial veins of the thoracoepigastric system. Management is observation and includes the use of warm, moist dressings and anti-inflammatory agents for symptomatic relief. The use of anticoagulation, antibiotics, or steroids is not indicated. Implant removal is not indicated in the absence of infection. Duplex ultrasonography is not required for management.

2015

40
Q

A 65-year-old woman comes to the office 1 month before a scheduled mastopexy. Annual mammography shows a 1.5-cm mass in the upper outer quadrant. Core needle biopsy is performed. Pathologic examination of excised tissue identifies papilloma without atypia. Which of the following is the most appropriate next step in management?

A) Bilateral breast sonography
B) Excisional biopsy of needle-localized area
C) Repeat annual mammography in 12 months
D) Repeat mammography at 6-month intervals for 1 year
E) Stereotactic vacuum-assisted biopsy

A

The correct response is Option B.

Percutaneous biopsy methods are commonly accepted for the initial evaluation of clinically occult breast lesions, although certain nonmalignant lesions pose dilemmas with respect to the most appropriate clinical management. Papillary lesions of the breast can either be benign or malignant, although differentiation is radiologically difficult. Moreover, it is difficult for pathologists to reliably distinguish among benign, atypical, and malignant papillary lesions on the limited fragmented tissue specimens they receive after needle sampling.

Previous studies have demonstrated high rates of ductal carcinoma in situ (11%) in patients diagnosed with benign papillomas by needle biopsy and who subsequently underwent a surgical excision, although conflicting data suggest an extremely decreased rate of malignancy when histology is benign on needle biopsy.

The management of benign papillary lesions is somewhat controversial. Although conservative follow-up with either yearly mammogram or short-interval follow-up may be appropriate for certain patients diagnosed with benign papilloma, certain features of this patient’s lesion make conservative follow-up inappropriate. Sonographic follow-up in a 65-year-old woman with mature breast parenchyma and a solid mammographically detected mass would not provide much additional information, and a repeat percutaneous biopsy, whether core needle or vacuum-assisted, would also not be effective. Given the size of the lesion and the age of the patient, surgical excision is warranted despite the lack of atypia on needle biopsy. Benign papillomas tend to be smaller than 1 cm and centrally located, whereas malignant lesions are more often greater than 1.5 cm and are peripherally located.

2015

41
Q

A 28-year-old woman desires augmentation mammaplasty with silicone implants. Physical examination shows tuberous breast deformity with an elevated inframammary crease. Sternal notch to nipple distance is 21 cm bilaterally. Nipple to inframammary crease distance is 3.5 cm bilaterally. Periareolar mastopexy with 350-mL silicone implants is planned. Which of the following operative plans will most effectively minimize the likelihood of a double-bubble deformity?

A) Lower the inframammary crease by 3 cm
B) Perform radial release of the lower pole breast fascia
C) Place implants in subparenchymal pocket
D) Reinforce the inframammary crease with acellular dermal matrix
E) Use highly cohesive gel implants

A

The correct response is Option B.

The tuberous breast is a developmental deformity characterized by a constricted inframammary fold, short nipple to inframammary crease distance, and both horizontal and vertical deficiencies. The pathophysiology of the tuberous breast predisposes the patient to develop a double-bubble deformity. In this patient, the inframammary crease must be lowered to accommodate the implant and improve the vertical skin deficiency. Radial release of the lower pole breast fascia is done with either a cautery or a knife. Multiple radial incisions are made, thereby allowing the tight crease to expand and decrease the chance for a double-bubble deformity.

Lowering the crease is necessary but will increase the chances of a double-bubble deformity. Subparenchymal implant placement and use of highly cohesive gel implants may help but are not the essential procedures required. The use of acellular dermal matrix can help secure the position of the inframammary crease in a patient who develops a double-bubble deformity secondary to an inferior migration of the implant below the inframammary crease. This does not apply in the patient described.

2015

42
Q

A 37-year-old woman comes to the clinic to be evaluated for augmentation mammaplasty to improve her breast shape. She is gravida 3, para 3, and breast-fed all of her children. On examination, she has decreased superior pole volume, and the distance from nipple to sternal notch is 28 cm. The nipple-areola complex is below the inframammary fold by 4 cm and is at the lower contour of the breast. Which of the following Regnault classifications of ptosis best describes these findings?

A) Grade I
B) Grade II
C) Grade III
D) Pseudoptosis

A

The correct response is Option C.

The Regnault classification of breast ptosis is based on the relationship of the nipple to the inframammary fold (IMF) and to the lower contour of the gland.

Pseudoptosis is the not true ptosis. In this situation, the nipple is above the level of the IMF but the breast parenchyma has descended below the IMF.

Grade I is minor ptosis with the nipple at the level of the IMF and above the lower contour of the gland.

Grade II is moderate ptosis with the nipple below the level of the IMF and above the lower contour of the gland.

Grade III is major ptosis with the nipple below the level of the IMF and at the lower contour of the gland.

2015

43
Q

A French woman, who underwent placement of Poly Implant Prothèse (PIP) gel implants in 2009, comes to the office for consultation because she had heard that the implants were filled with a nonmedical grade silicone. She reports that she has not had any problems with the implants, but would like to know the implications of retaining the implants and whether she should have them removed. This patient should be told that she is at increased risk for which of the following complications if she retains the implants?

A) Breast cancer
B) Cytotoxicity
C) Heavy metal poisoning
D) Implant rupture
E) Siloxane poisoning

A

The correct response is Option D.

The final report, in conjunction with the Department of Health in Australia, has shown a 2 to 6 times increased rupture rate in Poly Implant Prothèse (PIP) implants, which is detectable within 5 years of implantation. Increased levels of siloxane have been detected, but are not considered a health risk. No organic impurities have been detected and platinum levels are decreased in PIP gel compared with medical grade silicone. There is no increased breast cancer risk and no evidence of cytotoxicity. In the light of the increased rupture rate and the nonmedical grade nature of PIP silicone gel, the following recommendations were made:

all providers of breast implant surgery should contact any women who have or may have PIP implants, if they have not already done so, and offer them a specialist consultation and any appropriate investigation to determine if the implants are still intact;

if the original provider is unable or unwilling to do this, a woman should seek referral through her general practitioner to an appropriate specialist;

if there is any sign of rupture, she should be offered an explantation;

if the implants still appear to be intact, she should be offered the opportunity to discuss with her specialist the best way forward;

if, in the light of this advice a woman decides with her specialist that, in her individual circumstances, she wishes to have her implants removed, her health care provider should support her in carrying out this surgery. Where her original provider is unable or unwilling to help, the NHS will remove, but not normally replace, the implants;

if a woman decides not to seek early explantation, she should be offered annual follow up in line with the advice issued by the specialty surgical associations in January 2012. Women who make this choice should be encouraged to consult their doctor if they notice any signs of tenderness or pain, or swollen lymph glands in or around their breasts or armpits, which may indicate a rupture. At the first signs of rupture, they should be offered removal of the implants.

2015

44
Q

A 35-year-old woman comes to the office for consultation because she is dissatisfied with the appearance of her “deflated” and “saggy” breasts. Augmentation/mastopexy is planned. Compared with placement of the implant in the subglandular position, placement of the implant in the subpectoral space will preserve blood supply to the breast tissue and skin through which of the following arteries?

A) Internal thoracic
B) Lateral thoracic
C) Superficial superior epigastric
D) Thoracoacromial
E) Thoracodorsal

A

The correct response is Option D.

The perfusion of the nipple-areola complex is a major concern during breast procedures involving periareolar and intraparenchymal incisions. The nipple-areola complex has a very rich and overlapping perfusion through multiple sources. This fact allows the design of various pedicles to carry the nipple and areola with different techniques. The blood supply through the internal thoracic vessels reaches the breast, nipple, and areola through the intercostal perforators, which may be divided during both subpectoral and subglandular implant placement.

The location of the implant deep or superficial to the pectoralis muscle will not change the perfusion through the superficial epigastric vessels. The same is true for the blood supply through the lateral thoracic vessels. However, the flow through the thoracoacromial vessels to the breast parenchyma will be preserved by placement of the implant deep to the pectoralis muscle. Creation of a subglandular pocket above the muscle will interrupt the collaterals from the thoracoacromial vessels through the muscle to the parenchyma.

The thoracodorsal artery is not a major source of blood supply to the breast and the position of the implant will not affect it.

2015

45
Q

A 32-year-old woman is scheduled to undergo augmentation mammaplasty with highly cohesive, anatomically shaped, silicone-filled breast implants. She asks the surgeon about postoperative monitoring for implant rupture. This patient should be counseled that, according to FDA recommendations, postoperative monitoring for rupture most appropriately includes which of the following?

A) Manual examination 3 years postoperatively, then annually thereafter
B) MRI screening 2 years postoperatively, then every 3 years thereafter
C) MRI screening 3 years postoperatively, then every 2 years thereafter
D) Ultrasonography screening 2 years postoperatively, then every 3 years thereafter
E) Ultrasonography screening 3 years postoperatively, then every 2 years thereafter

A

The correct response is Option C.

Diagnosis of rupture is difficult by physical examination alone, which is why the majority of ruptures are silent. Subsequent MRI screening for silent rupture is recommended initially 3 years postoperatively, then every 2 years thereafter.

Highly cohesive, anatomically shaped, silicone-filled breast implants combine the “gummy bear” silicone with an anatomical shape, in which inferior pole projection is higher than the superior pole projection. In studies of Allergan’s Natrelle 410 breast implants (the “Pivotal Study,” the 410 Swedish MRI study, and the 410 European MRI study) approximately 3 in 100 women had silent ruptures.

Cohesive gel is still subject to rupture, because rupture occurs when the shell fails. In cohesive implants, however, as opposed to noncohesive implants, the rupture rarely becomes extracapsular.

2015

46
Q

A 45-year-old woman comes to the office 10 years after undergoing subglandular implantation of textured silicone implants for augmentation mammaplasty. Physical examination shows swelling of the left breast. She is concerned about cancer. Increased incidence of which of the following malignancies is associated with breast implants?

A) Acute myeloid leukemia
B) Anaplastic large cell lymphoma
C) Angiosarcoma
D) Infiltrating ductal carcinoma
E) Malignant fibrous histiocytoma

A

The correct response is Option B.

Several reports have suggested an association between breast implants and anaplastic large cell lymphoma (ALCL), which is an extremely rare malignancy. In these cases, ALCL has usually occurred several years after implantation as swelling or a mass around the implant and is often associated with a periprosthetic seroma. Treatments have included capsulectomy with implant removal and chemotherapy and/or radiation therapy, though there is no defined consensus regimen. Despite evidence of an increased risk of ALCL in breast implant patients, the absolute risk remains extremely low.

Several large epidemiologic studies have demonstrated a similar or lower incidence of breast cancer (infiltrating ductal carcinoma) among patients who have undergone prosthetic augmentation mammaplasty surgery compared with those who have not. Most cases of ALCL have been in textured implants.

Angiosarcoma and malignant fibrous histiocytoma are two sarcomas that may arise in the breast. Angiosarcoma may be caused by radiation therapy for breast cancer. Neither of these sarcomas has been associated with breast implants.

Acute myeloid leukemia may be associated with radiation treatment to the breast but has not been associated with breast implants.

2014

47
Q

A 33-year-old woman with no family history of breast cancer undergoes bilateral augmentation mammaplasty with 300 mL of autologous fat per breast. Six months later, she has onset of pain in the right breast. Mammography shows linear clustered microcalcifications in the lower inner quadrant of the right breast, small lipid cysts bilaterally with scattered dystrophic rod-like calcifications in the upper outer quadrants bilaterally, and heterogeneity of the pectoral muscles. Which of the following is the most appropriate next step in management?

A) Baseline mammography between ages 35 and 40 and yearly thereafter
B) Core needle biopsy of the bilateral upper outer quadrants
C) Core needle biopsy of the right lower inner quadrant
D) Repeat mammography at 6 months and 12 months
E) Repeat mammography in 1 year

A

The correct response is Option C.

Augmentation mammaplasty with autologous fat transfer has become an increasingly popular option for patients desiring modest volumetric improvement. Despite its popularity, there is still some concern regarding its safety and efficacy. ASPS offered guidelines on fat grafting for reconstructive procedures of the breast in 2009. However, caution is recommended in the setting of cosmetic procedures because the impact on radiologic changes in follow-up is still uncertain to date.

Fat necrosis is a nonspecific histologic finding most commonly resulting from surgery, trauma, or radiation therapy. It is common after fat transfer procedures, though often is clinically occult, and detected through follow-up mammography. The mammographic images of fat necrosis range from lipid cysts to findings that are suspected for malignancy such as clustered microcalcifications or spiculated masses. The most frequent mammographic finding in the breast parenchyma after augmentation mammaplasty with fat transfer is bilateral scattered microcalcifications followed by radiolucent oil cysts with or without microcalcification. Microcalcifications represent an evolution in the mammographic appearance of fat necrosis and are usually not present in early postoperative screening, but rather are a relatively late finding that is present months to years after the inciting trauma.

It is imperative that radiologists distinguish between benign and suspected microcalcifications in order to minimize the number of postoperative biopsies and frequent follow-up imaging. Although round, spherical, punctuate, and diffusely scattered calcifications are typical of benign processes, cluster, branching microcalcifications can be indicative of a malignant process and should be worked up. For this 33-year-old patient with no baseline mammography and a suspected lesion within 6 months of the procedure, routine or short-interval mammographic screening is not appropriate. A biopsy of the suspected area is required, and this patient should undergo a core needle biopsy of the clustered microcalcifications of the right breast, while the more benign-appearing calcifications within the upper outer quadrants can be observed.

2014

48
Q

A 35-year-old woman with tuberous breast deformity is scheduled to undergo augmentation/mastopexy. A smooth, round, cohesive gel implant will be used. This patient is at higher risk for which of the following complications when compared with augmentation/mastopexy performed on a patient without a tuberous breast?

A) Capsule contracture
B) Double bubble
C) Hematoma
D) Nipple-areola depigmentation
E) Rippling

A

The correct response is Option B.

The classic features of a tuberous breast deformity include a constricted base with a high inframammary crease and herniation of breast parenchyma into the nipple-areola complex producing a large-diameter areola. Variable extent of micromastia is associated as well as breast asymmetry. When a patient has a high and tight inframammary crease, this crease must be released to accommodate an implant and allow correction of the deformity. If this native crease does not fully expand, then a double bubble will occur. Over time, the lower pole skin stretches in response to the implant and this double bubble often improves spontaneously. The incidence of capsule contracture, hematoma, nipple-areola depigmentation, and rippling should be similar to a patient who undergoes periareolar augmentation/mastopexy without a tuberous breast.

2014

49
Q

A 28-year-old woman is scheduled to undergo vertical mastopexy. She has no history of previous breast surgery. A superior pedicle technique is planned. Which of the following is the dominant blood supply for this pedicle?

A) Deep branches of the internal mammary artery from the fourth interspace
B) Deep branches of the internal mammary artery from the fifth interspace
C) Superficial branches of the internal mammary artery from the second interspace
D) Superficial branches of the internal mammary artery from the fourth interspace
E) Superficial branches of the lateral thoracic artery

A

The correct response is Option C.

The breast receives its arterial blood supply from multiple sources, and this fact is used to design multiple pedicles for the nipple-areola complex that can work reliably for both mastopexy and reduction mammaplasty procedures.

The superior pedicle receives its arterial blood supply primarily from the internal mammary branch from the second interspace. It is usually about 1 to 2 cm below the surface of the skin just medial to the breast meridian as it approaches the areola and may be localized with a handheld Doppler device during preoperative planning.

The inferior pedicle and central pedicle designs are primarily supplied by branches of the internal mammary system from the fourth interspace. Additionally, there is some accessory input from the intercostal branches at the level of the inframammary fold with the inferior pedicle design. These secondary vessels are typically interrupted in a central pedicle operation.

The medial pedicle design receives its arterial input mainly from the third superficial branch of the internal mammary artery. This vessel may be damaged by previous augmentation mammaplasty.

The lateral pedicle design receives its arterial supply from superficial branches of the lateral thoracic artery.

2014

50
Q

An otherwise healthy 40-year-old woman comes to the office for augmentation mammaplasty. Mammography 6 months ago showed no abnormalities. Family history is negative for breast cancer. She wants to know if silicone gel implants are safe and what she should do after the procedure to monitor the implant for evidence of rupture. According to the current federal guidelines, which of the following is the most appropriate recommendation to this patient regarding surveillance?

A) MRI 3 years after implantation and every 2 years thereafter
B) MRI every 10 years
C) MRI if symptoms such as chronic myalgia and fatigue develop
D) Yearly mammograms
E) Yearly MRI

A

The correct response is Option A.

Evidence-based data to confirm the validity of screening patients with silicone implants are lacking. In 2011, the FDA issued recommendations for physicians on the use of silicone gel-filled implants. Recommendations included providing copies of educational brochures, giving appropriate informed consent, maintaining medical vigilance, and reporting adverse events. It also suggested that patients undergoing augmentation mammaplasty get an MRI 3 years after implant placement and every 2 years thereafter. The purpose of these recommendations is not to replace routine cancer surveillance.

2014

51
Q

A 30-year-old woman comes to the office for augmentation mammaplasty and mastopexy after a 50-lb (23-kg) weight loss. She wears a size 38B brassiere. Physical examination shows grade II ptosis and a sternal notch to nipple distance of 26 cm bilaterally. Simultaneous augmentation mammaplasty with short-T mastopexy using smooth saline-filled breast implants that will be implanted in a dual-plane configuration through an inframammary incision is planned. Which of the following factors puts this patient at highest risk for reoperation?

A) Inframammary implant insertion route
B) Presence of breast ptosis
C) Use of drains
D) Use of saline implants
E) Use of smooth-walled implants

A

The correct response is Option B.

It has long been realized that combination augmentation mammaplasty operations are more difficult and have a higher revision rate than either operation alone. A recent review of 177 primary augmentation mammaplasty cases found that, of the factors listed, preexisting breast ptosis and simultaneous mastopexy were both linked to a higher rate of reoperation when possible contributing factors were statistically analyzed. Furthermore, increasing grades of breast ptosis were linked with increasingly higher reoperation rates.

Although incision site for augmentation mammaplasty has been markedly linked to the rates of capsular contracture, inframammary incisions have been shown in at least two studies to date to have the lowest rate of capsule formation, with periareolar and transaxillary incisions showing 5 to 10 times higher rates of capsule-related complications.

2014

52
Q

A 30-year-old woman comes to the office because of a 3-week history of unilateral swelling of the left breast. She underwent subglandular placement of textured silicone breast implants 4 years ago. She has had no trauma, fevers, or chills. A 1-week course of an oral antibiotic prescribed by her family physician has failed to resolve the swelling. On physical examination, the left breast is 300 to 400 mL larger than the right breast. No other abnormalities are noted. Ultrasonography report shows seroma and results are negative for hematoma or mass. Which of the following is the most likely diagnosis in this patient?

A) Anaplastic large cell lymphoma
B) Double capsule phenomenon
C) Giant fibroadenoma of the breast
D) Hematoma due to capsule tear
E) Periprosthetic abscess

A

The correct response is Option B.

The combination of late-onset swelling without signs of periprosthetic infection (fever, cellulitis), no history of trauma, and a negative ultrasonography suggests late-onset seroma, as can occur with a double capsule phenomenon. Late seromas occur as a complication in about 1% of reported breast implant series. This issue seems to be more common in the setting of textured implants, particularly those implants manufactured with an aggressive texturing process. At surgery, a capsule layer is seen lining the pocket, which often contains a substantial volume of serosangineous seroma fluid and a textured implant coated in a tight second capsule at the center of the pocket. Double capsule has been reported in both the subglandular and submuscular positions. A giant fibroadenoma of the breast would have a dominant mass, distortion of the breast shape, and would be visible on ultrasonography. Abscess would be likely to occur with fever, chills, and cellulitis of the breast. Hematoma of this size would be likely to have a history of trauma, breast pain, and external bruising. Although anaplastic large cell lymphoma is a possibility in the differential of late-onset seromas, it is a rare disorder. Seroma fluid, obtained either by ultrasound-guided aspiration or at the time of open surgery, should be sent for cytologic examination and immunohistochemistry to rule out this rare possibility.

2014

53
Q

A 53-year-old woman comes to the office for evaluation of breast asymmetry. Reduction of the left breast and augmentation of the right breast with implant and autologous fat transfer are planned. She is concerned about fat injection and cancer risk. Which of the following is the most appropriate response regarding mammographic changes after fat transfer?

A) Calcifications warranting biopsy are more likely on the fat transfer side
B) Calcifications warranting biopsy are more likely on the reduction side
C) Masses requiring biopsy are more likely on the reduction side
D) Scarring will be decreased on the reduction side
E) There are no differences between mammographic findings in fat transfer and reduction

A

The correct response is Option C.

Fat transfer to the breast remains a controversial procedure. There are some concerns about the oncologic safety of fat transfer, and for this reason some authors do not recommend fat transfer in patients with a history of cancer. Another concern about fat transfer is the potential difficulty in screening for malignancy. Rubin, et al. compared mammographic changes after fat transfer with changes after reduction mammaplasty. In this blinded study, radiologists reviewed pre- and postoperative mammograms of patients who had undergone augmentation and fat transfer and reduction mammaplasty. In the reduction cohort, masses requiring biopsy and scarring were more common; other abnormalities, including oil cysts, benign calcifications, and calcifications requiring biopsy showed no differences between the groups.

2014

54
Q

A 47-year-old woman who underwent bilateral augmentation mammaplasty with silicone implants to treat mammary hypoplasia 17 years ago is evaluated because of worsening pain, firmness, and distortion of her breasts. Which of the following diagnostic evaluations is most sensitive for evaluating this patient’s silicone breast implants?

A) Breast thermography
B) CT scan
C) Mammography
D) MRI
E) Ultrasonography

A

The correct response is Option D.

MRI scan would be the most sensitive and specific method for detection of silent rupture of a silicone breast implant in this patient. Classic MRI findings indicating rupture include the linguini sign or the teardrop sign. Current FDA recommendations are to obtain MRI screening for silent rupture three years after placement of silicone implants and every two years after that.

CT scanning can show findings similar to those seen with MRI, but CT involves ionizing radiation, which can be harmful. CT has not been proven to be as sensitive as MRI in evaluating silicone breast implant rupture.

Ultrasonography is a less costly method of implant evaluation but this method is highly operator-dependent. In asymptomatic women, a subsequent MRI scan is generally needed to confirm a positive ultrasound screen.

Mammography is indicated for screening for breast cancer but not for implant rupture.

Breast thermography utilizes digital infrared imaging to evaluate metabolic activity and vascular circulation of the breast to look for suspicious signs of breast cancer. It is not effective in the evaluation of silicone breast implant rupture.

2019

55
Q

A 34-year-old woman with a history of grade I breast ptosis who is 6 years status post-augmentation mammaplasty with subglandular gel-filled implants returns to the clinic. Physical examination shows normal-appearing breasts, but there is mild firmness on palpation. Which of the following Baker grades best describes these findings?

A) Grade I
B) Grade II
C) Grade III
D) Grade IV

A

The correct response is Option B.

Many classification systems have been used to evaluate the severity of breast capsular contracture, which occurs when the peri-implant capsule undergoes fibrotic change. The most widely employed assessment tool remains the Baker grading system, which takes into account patient signs and symptoms. According to the Baker classification, only the highest degrees of contractures (grades III and IV) require surgical treatment. The descriptors for each grade are listed here:

Grade I: the breast is soft and appears normal in size and shape
Grade II: the breast is a little firm and appears normal
Grade III: the breast is firm and appears abnormal
Grade IV: the breast is firm, appears abnormal, and is painful

Studies note decreased relative risk for Baker grade III to IV capsular contracture in patients who undergo primary breast augmentation through an inframammary fold incision, subpectoral pocket placement, and textured implants. There is an increased relative risk for capsular contracture when patients undergo a periareolar or axillary incision and subglandular placement of smooth implants.

2019

56
Q

A 36-year-old woman is evaluated because of spontaneous galactorrhea 6 days after undergoing augmentation mammaplasty. Which of the following factors most likely contributed to this outcome?

A) Inframammary placement of the incision
B) Subglandular versus dual-plane position of the device
C) Surgical interruption of the intercostal nerves
D) Use of silicone versus saline breast implants

A

The correct response is Option C.

Although no one knows exactly what leads to postoperative galactorrhea, it is observed to occur more often in parous women and theorized to occur due to a combination of factors which simulate suckling or change in the innervation of the chest wall and nipple-areola complex. This would include increased tissue pressure related to the implant placement and interruption of intercostal nerves. No relationship has been identified between incision placement (peri-areolar, inframammary, transaxillary, or even peri-thelial) and postoperative galactorrhea. Similarly no relationship has been identified between device positioning (dual-plane, subglandular, and submuscular) and postoperative galactorrhea. Again, no relationship has been observed in implant type, saline versus silicone, and postoperative galactorrhea.

2019

57
Q

A 40-year-old woman comes to the office for consultation on an augmentation mastopexy 2 years after giving birth to her second child. She is back to her pre-pregnancy weight. Physical examination shows involutional changes contributing to a deflated appearance of the breasts. This appearance is most likely due to a histologic decrease in which of the following?

A) Area composed of stromal matrix
B) Number of differentiated lobules
C) Thickness of dermis
D) Thickness of pectoralis muscle
E) Volume of adipose tissue

A

The correct response is Option B.

Postpartum involutional changes can manifest clinically as breasts that appear deflated, commonly due to a loss of volume and skin that has been stretched. On a histologic level, these clinical manifestations occur due to a decrease in the number and area of differentiated lobules that were enlarged and specialized for milk production. As this occurs, it is hypothesized that the lobular area is then replaced by stromal matrix and eventually fat. Involutional changes do not refer to changes in the dermis, pectoralis muscle or chest wall structures.

2019

58
Q

A 26-year-old healthy woman comes to the office for consultation because she has constant pain 1 year after undergoing augmentation mammaplasty by another surgeon. She reports that he “botched” her surgery and that she is considering taking legal action against him. On physical examination, the breasts are quite firm and mildly tender. The relatively immobile subglandular implants are high on the chest wall. There are no overlying skin changes. Which of the following is the most appropriate response by the surgeon in this scenario?

A) Decline to establish care
B) Follow-up visit in one year
C) Perform a diagnostic intercostal nerve block
D) Prescribe a course of oral corticosteroids
E) Recommend surgical intervention

A

The correct response is Option E.

This patient has developed Baker grade IV capsular contracture as evidenced by hard, painful breast implants that are malpositioned. This is a known complication of augmentation mammaplasty, and the patient should be informed that it is treatable with another surgery. It would be reassuring to the patient to hear that it is a known post-operative complication that happens not uncommonly, and that it is unlikely the other surgeon directly did anything to cause this. Thoughtful analysis and contextualization are helpful in high-tension consultations such as these.

Providing the patient with a malpractice attorney’s contact information may be what she thinks she wants, but diffusing the situation is best for all involved. Dismissing the patient’s concerns outright without diagnosing her and suggesting a course of treatment would not be helpful. The new surgeon may even take it a step further by offering to speak with her previous surgeon to discuss the patient’s concerns and the findings seen during consultation. The patient may refuse to allow this, and she may have lost faith in her other surgeon, but at least offering to speak with the other surgeon is prudent and may restore the relationship between this patient and her surgeon. A pain management specialist and physical therapy may help somewhat with symptoms, but her problem ultimately requires and should respond well to a surgical solution.

2019

59
Q

A 39-year-old woman comes to the office with a 6-month history of progressive firmness and superior fullness of the left breast. History includes bilateral augmentation mammaplasty with textured saline implants placed in a submuscular dual-plane pocket 15 years ago. On physical examination, the left breast appears larger and firmer with more upper pole fullness in comparison with the right breast. Which of the following is the most appropriate next step?

A) Capsulectomy and pocket change
B) Mammography
C) 3-Month trial of montelukast (Singulair)
D) MRI
E) Ultrasonography

A

The correct response is Option E.

Breast implant patients who present with late-onset enlargement of one of their breasts require evaluation for breast implant-associated anaplastic large cell lymphoma (BIA-ALCL). This disease usually presents with spontaneous onset of peri-prosthetic fluid. A late-onset seroma is usually accepted as occurring 1 year after surgery; however, they have presented as early as 4 months. It is often difficult to determine if late-onset firmness of the implant is secondary to fluid, capsular contracture, or both.

The initial workup should begin with an ultrasound to evaluate for peri-prosthetic fluid or capsular mass. If fluid is present, it should be sent for cytology, flow cytometry with immunohistochemistry looking for expression of T-cell CD30 cell surface protein.

BIA-ALCL is overwhelmingly associated with textured implants. It is important to remember that BIA-ALCL is extremely rare and that most patients presenting with a late seroma will not have lymphoma, but will have peri-prosthetic fluid from the textured surface pulling away from the capsule and forming a double capsule. The treatment for localized BIA-ALCL is bilateral total capsulectomy and explantation. Treatment for the more likely double capsule or capsule contracture is capsulectomy and pocket change; however, surgery is not indicated until the diagnosis is made.

Montelukast is a leukotriene antagonist that can inhibit the inflammatory cascade thought to be involved with capsular contracture. It seems to be more useful in patients with capsular contracture less than grade III. There is no consensus on its use or effectiveness.

The sensitivity and specificity of ultrasound for detecting a seroma has been equal or better than MRI or 3D mammography. After diagnosis of ALCL, MRI and PET scanning may be indicated. If the implants were silicone gel, MRI would be indicated to evaluate for implant rupture; however, ultrasound would still be recommended for seroma evaluation and aspiration.

2019

60
Q

An otherwise healthy 25-year-old woman is evaluated and scheduled for augmentation mammaplasty with silicone gel implants. Which of the following is most accurate regarding breast implant-associated anaplastic large cell lymphoma (BIA-ALCL)?

A) All late breast implant-associated seromas should be evaluated
B) BIA-ALCL is most often associated with an aggressive clinical course
C) BIA-ALCL is most often associated with smooth implants
D) It is not necessary to include BIA-ALCL in a standard breast augmentation/reconstruction consent
E) Knowledge about BIA-ALCL’s cause is based on strong evidence-based studies

A

The correct response is Option A.

Due to the potential critical relevance of breast implant-associated anaplastic large cell lymphoma (BIA-ALCL) and its tendency to present as a late seroma, correct diagnostic pathways should be carried out on all late breast implant-associated seromas to include cytologic examination and, if indicated further, fine-needle aspiration, flow cytometry, and immunohistochemistry for CD30. BIA-ALCL is a critical outcome in implant-associated breast augmentation/reconstruction. It is most commonly confined to peri-implant seroma fluid and follows a nonaggressive course amenable to implant and capsule removal, although there are aggressive variants. Discussion about this condition should be part of the consenting process for all breast implant cases. Finally, to this point evidence on cause is based on very low-evidence studies. ALCL is most often associated with textured implants.

2019

61
Q

A 55-year-old woman who underwent augmentation mammaplasty with retro-pectoral smooth saline implants 18 years ago comes to the office because she is dissatisfied with her breast shape. Physical examination shows glandular ptosis hanging off the implants. She has Grade I capsules. Which of the following is the most appropriate procedure to correct this patient’s deformity?

A) Implant exchange alone
B) Implant exchange with capsulectomy
C) Implant exchange with mastopexy
D) Implant exchange with suture plication of the expanded inferior pocket
E) Site change to subglandular placement

A

The correct response is Option C.

The described patient has a “snoopy nose deformity” or “waterfall breast deformity,” with the ptotic breast hanging off of the implant. There is no pocket expansion. The implants have stayed in their original position while the native breast tissue has become ptotic with time and gravity. This is not superior malposition due to capsular contracture; both breasts are soft. Correction of this problem is best performed with an appropriately chosen form of mastopexy. In this case, replacement of the implants would also be performed because of their age.

Implant exchange alone would not correct the ptotic breast. Capsulectomy is not indicated, since the breasts are soft, and no capsule is noted clinically. Similarly, suture plication of the pocket is not required, since the inframammary fold is in the correct position, and no second fold is seen. Site change would not correct the patient’s grade III ptosis, but it may be used to correct this problem in cases without significant ptosis.

2019

62
Q

A 28-year-old postpartum woman comes to the office for evaluation of breast asymmetry with pain and enlargement of the right breast for 2 months. Medical history includes augmentation mammaplasty 4 years ago. She denies fever or chills. She was previously breast-feeding but stopped this 1 month ago. Physical examination shows the right breast is significantly larger than the left breast. A well-healed peri-areolar incision is present and no evidence of infection is noted. Ultrasound shows a complex cyst, which yields 150 cc of milky fluid. A drain is placed. The most appropriate next step is administration of which of the following medications?

A) Bromocriptine
B) Cephalexin
C) Fluconazole
D) Prolactin
E) Trimethoprim-sulfamethoxazole

A

The correct response is Option A.

This postpartum patient is presenting with a symptomatic galactocele after breast-feeding. Galactoceles are benign breast cysts containing milk. They typically occur in women of childbearing age in the setting of active lactation, recent pregnancy, or the use of hormonal medications such as oral contraceptives. The galactocele is thought to occur from ductal obstruction. Although the presence of a breast implant and the respective pocket placement is unknown to have an effect on the development of galactoceles, there is some thought that peri-areolar incisions may contribute to the ductal obstruction. There are, however, documented cases of post-augmentation galactocele without peri-areolar incisions.

Treatment for a galactocele is typically medical with the initiation of oral bromocriptine. Bromocriptine is a dopamine receptor agonist and causes inhibition of prolactin secretion, which is the primary hormone responsible for milk production. Dosage is titrated to effect. Incision and drainage of the cyst, particularly in the setting of implants, is often performed as well to rule out the possibility of infection.

Cephalexin and trimethoprim-sulfamethoxazole are antibiotics and are not indicated in this case because there is no active infection. Fluconazole is indicated for the treatment of fungal infections. Prolactin would actually stimulate milk production and would worsen the patient’s symptoms.

2019

63
Q

A 54-year-old woman with a history of augmentation mammaplasty with textured silicone implants has histologic confirmation of breast implant–associated anaplastic large cell lymphoma (BIA-ALCL). MRI and PET scans show no associated masses, with activity localized to the periprosthetic seroma. Which of the following is the most appropriate next step in management of this patient?

A) Anterior capsulectomy with removal of the implants bilaterally
B) Complete capsulectomy with removal of the implant on the affected side
C) Partial capsulectomy with replacement of the implant
D) Removal of the textured implant and replacement with a smooth implant
E) Sealing of the seroma cavity with fibrin glue

A

The correct response is Option B.

Breast implant–associated anaplastic large cell lymphoma (BIA-ALCL) is a rare peripheral T-cell lymphoma that has been increasingly recognized as a serious, albeit uncommon, complication associated with the use of textured breast implants. Since the initial case report in 1996, there have been continually increasing reported cases of this rare malignancy and according to the most recent data available, the lifetime risk of association between breast implants and BIA-ALCL is between 1 in 1000 to 1 in 30,000 with the ASPS recognizing nearly 200 cases in the US and nearly 500 cases worldwide.

BIA-ALCL patients typically present with a spontaneously occurring periprosthetic fluid collection or capsule-associated mass approximately 10 years following implantation of the breast implant. To date, all cases have had some association with a textured device. Initial workup includes ultrasound for evaluation of a periprosthetic fluid collection or mass. Periprosthetic fluid collections should undergo fine-needle aspiration in the clinic or ultrasound-guided aspiration by interventional radiology if there is concern for trauma to the implant while masses require tissue biopsy. Specimens should be sent for cytology with immunohistochemistry and flow cytometry for T-cell markers, specifically CD30 cell surface protein. A recent systematic review revealed that 66% of BIA-ALCL patients presented with isolated late-onset seroma while only 8% presented with an isolated new breast mass.

National Comprehensive Cancer Network (NCCN) guidelines for treatment of BIA-ALCL recommend complete removal of the lymphoma (fluid and/or mass), complete capsulectomy, and removal of the implant. More advanced disease may require chemotherapy, radiotherapy, and/or lymph node dissection. Although some surgeons advocate removal of the contralateral breast implant as approximately 4.6% of cases have demonstrated incidental lymphoma in the contralateral breast, this recommendation is controversial. The official NCCN guidelines for treatment only recommend consideration of contralateral breast implant removal but this is not mandated.

2019

64
Q

A 27-year-old woman who underwent augmentation mammaplasty with 325-mL textured prostheses one year ago comes to the clinic because her breasts look asymmetric and feel hard. Physical examination shows firm asymmetric breasts with palpable capsules. No pain, signs of skin infection, hematoma, or seroma are observed. Hypertrophic scars are seen on the inframammary fold of both breasts. Which of the following factors is the most likely cause of capsular contracture in this patient?

A) Implant size
B) Patient history of hypertrophic scarring
C) Subclinical infection with biofilm formation
D) Submuscular positioning of the implants
E) Textured implants

A

The correct response is Option C.

On the basis of her clinical presentation, this patient is experiencing Baker Grade III capsular contracture. Capsular contracture is the most common complication after breast implant placement. This is a multifactorial complication; however, only subclinical infection with biofilm formation has a clear correlation with a higher degree of capsular contracture.

Implant size is not directly associated with an increased risk for clinically significant capsular contracture, and it has been established that textured implants are associated with a decreased risk. There is no clear evidence of a relation between a patient’s tendency to scar and an increased risk for capsular contracture. It is accepted that submuscular placement leads to lower rates of capsular contracture than the subglandular technique.

2020

65
Q

A healthy 45-year-old woman presents for consultation for mastopexy. Examination of the breasts shows grade II ptosis, large areolae, and dense breast tissue. No prior surgical scars are noted. An inverted T mastopexy is planned. On the day of surgery, the angle of the vertical limbs has to be marked wider than anticipated because of the large areolae. The large angle of divergence of the vertical limbs most likely increases the risk for which of the following?

A) Lower pole deformity
B) Nipple-areola malposition
C) Nipple-areola slough
D) Parenchymal fat necrosis
E) Pedicle overresection

A

The correct response is Option A.

For a patient undergoing a full-scar, inverted T skin resection as part of mastopexy, large areolae may require that the vertical limbs diverge more widely than would otherwise be necessary to tighten excess skin. This can create lower pole deformities such as flattening or boxiness.

The position of the nipple-areola complex is usually set at the Pitanguy point—the level determined by transposition of the inframammary crease onto the breast—and is not affected by large areolae in a patient who is a candidate for inverted T mastopexy. Necrosis of tissues such as the nipple-areola complex or breast parenchyma relate to surgical technique and preservation of blood supply to these areas. Pedicle overresection can lead to nipple-areola necrosis and is a result of poor surgical technique in developing the pedicle; pedicle design and resection are independent from the design of skin resection, as seen in this patient.

2020

66
Q

A 32-year-old woman is evaluated 4 years after undergoing bilateral augmentation mammaplasty with 375-mL, textured, shaped gel implants. She is concerned because over the past 3 months her left breast has grown one cup size larger than her right. On examination, there is no erythema or palpable mass. The left breast is much larger and firmer than the right. The patient started taking an oral contraceptive 6 months ago. Which of the following is the most likely diagnosis?

A) Breast implant–associated anaplastic large cell lymphoma
B) Capsular contracture
C) Double capsule
D) Drug-induced breast hypertrophy
E) Phyllodes tumor

A

The correct response is Option C.

This patient presents with a late seroma (more than 1 year after surgery) following augmentation mammaplasty with textured implants. The most common reason for this late seroma is a benign process related to the textured implant. A double capsule forms when the textured implant surface breaks away from its attachment to the breast parenchyma and forms a double capsule, which can then fill with blood or fluid. This phenomenon can be caused by a known trauma or in the course of daily living.

The possible etiologies for any late seroma include trauma, infection, inflammation, and malignancy. The first step in evaluation is ultrasound and aspiration of the fluid. The fluid should be evaluated for tumor markers (flow cytometry, CD30 T-cell surface protein) and sent for cytology and bacteriology.

This patient could have breast implant–associated anaplastic large cell lymphoma (BIA-ALCL). BIA-ALCL is rare, and it is highly unlikely that this patient has this lymphoma. Nonetheless, it must be ruled out. All patients presenting with a late seroma (more than 1 year after surgery) need to be evaluated for tumor markers. If the patient tests positive, full oncologic evaluation is needed.

Capsular contracture can cause hardening and deformity of the breast; however, it will not cause breast enlargement. A phyllodes tumor presents as a localized breast mass that can grow rapidly. This patient has no palpable masses.

Oral contraceptives can cause unilateral breast enlargement, but this patient is presenting with massive breast enlargement 6 months after starting birth control pills.

2020

67
Q

A 45-year-old woman is evaluated because of asymmetry of the chest during physical activity. Medical history includes bilateral augmentation mammaplasty with silicone implants approximately 10 years ago. She recently began an aggressive exercise regimen that includes long-distance running and weight training. The patient’s major aesthetic concern is that the implants distort the appearance of her breasts during weight training exercises. Which of the following is the best treatment for this patient?

A) Remove and replace the implants with more cohesive silicone gel implants
B) Remove and replace the implants with saline implants
C) Remove the implants and insert them in the subglandular plane
D) Remove the implants and insert them in the subpectoral plane
E) Remove the implants, perform bilateral capsulectomy, and insert new silicone implants

A

The correct response is Option C.

Muscle contraction deformity and implant displacement are corrected with the removal of the implant and placement into a newly created subglandular plane from the current subpectoral plane. The deformity described is typical in patients involved in heavy weight training in which the pectoralis muscle is repeatedly activated.

Moving the implants from the subpectoral plane into the subglandular plane will compress the subpectoral pocket, thus preventing movement into the previous pocket. In addition, the pectoralis muscle may be resuspended to prevent fluid accumulation.

Removal and replacement of the implants with more highly cohesive implants may improve any rippling that may be seen, but it will not improve the animation deformity. Removal and replacement with saline implants will not make any significant improvements in the muscle-induced deformity. While capsulectomy may improve capsular contraction, it does not address the deformity cause by repeated pectoralis activation.

2020

68
Q

A 31-year-old woman with micromastia comes to the office to discuss bilateral augmentation mammaplasty. Which of the following surgical plans will most likely minimize the risk for breast implant–associated anaplastic large cell lymphoma?

A) Highly cohesive silicone gel
B) Nipple shields
C) Saline implants
D) Smooth wall implants
E) Subpectoral pocket

A

The correct response is Option D.

It is extremely rare, if ever seen, to have a diagnosis of breast implant–associated anaplastic large cell lymphoma (BIA-ALCL) in a smooth implant–only case. In those BIA-ALCL cases with smooth wall implants, the patient’s past surgical history revealed the use of textured implants (where adequate documentation was available).

Aggressive texturing (macrotexturing) is associated with a higher risk for BIA-ALCL. One theory is related to increased surface area and therefore higher numbers of bacteria.

The most common presentation for BIA-ALCL is a patient presenting with a late seroma (usually greater than one year). Workup requires aspiration of the seroma followed by cytologic evaluation of the fluid and flow cytometry looking for CD30 T-cell surface protein, which is an ALCL tumor marker.

Highly cohesive silicone gel, saline implants, and the type of implant pocket do not significantly impact the incidence of BIA-ALCL.

Nipple shields and antibacterial irrigation can lower the incidence of bacterial contamination and MAY play a part in reducing biofilm, capsule contracture, and possibly BIA-ALCL. This impact on BIA-ALCL is unknown and being investigated. There are multiple documented cases of BIA-ALCL in patients with textured implants who underwent antibacterial irrigation to reduce bacterial contamination. Nonetheless, avoiding textured implants is the most likely strategy for minimizing BIA-ALCL.

2020

69
Q

A 23-year-old woman comes to the office because she is interested in augmentation mammaplasty. Physical examination shows a right-sided sunken anterior chest wall, hypoplasia of the right breast with a superiorly placed nipple-areola complex, normal pectoralis muscle, and normal sternal position. This patient most likely has which of the following congenital deformities?

A) Amastia
B) Anterior thoracic hypoplasia
C) Pectus carinatum
D) Pectus excavatum
E) Poland syndrome

A

The correct response is Option B.

Amastia refers to an uncommon developmental condition in which the breast and nipple are absent. Some women are immediately given the diagnosis of Poland syndrome or pectus deformity when they exhibit abnormalities of the anterior chest wall. Poland syndrome involves an abnormal pectoralis muscle while pectus deformities do not. Pectus deformities involve alteration in the appearance or location of the sternum and its costal attachments. Another less commonly realized diagnosis is that of anterior thoracic hypoplasia in which patients share the same characteristics of unilateral sunken anterior chest wall, hypoplasia of the breast, superiorly placed nipple-areola complex, normal pectoralis muscle, and normal sternal position.

2020

70
Q

A 45-year-old woman who underwent bilateral breast augmentation mammaplasty returns to the office after a motor vehicle accident with deployment of airbags. Physical examination shows point tenderness over the chest with an obvious “seat belt” sign along the left breast. The immediate diagnostic workup of the implants shows no rupture. Six months later, the patient returns with distortion of the left breast over the implant with a cleft formation. Which of the following is the most appropriate next step in assessing the integrity of the implants?

A) Chest x-ray study
B) CT scan with intravenous contrast
C) Mammography
D) MRI
E) Ultrasound

A

The correct response is Option D.

The patient has a late presentation of seat belt syndrome, which requires an MRI to assess the integrity of the breast implants. Patients with seat belt syndrome may present with a cleft or a mass. It is important to rule out invasive ductal carcinoma located in the line of the diagonal contracture. An intracapsular seroma can form gradually over time as well. Reconstructive options include unilateral capsulectomy and implant exchange.

Chest x-ray study can be used in a more immediate setting to rule out any bony injury to the chest wall. Mammography is an appropriate choice in patients who have a palpable mass in an initial assessment; however, this does not rule out implant rupture. In the event that there is an expanding breast in the immediate setting, CT scan with intravenous contrast can be used to rule out possible arterial extravasation or pneumothorax. Ultrasound can be used to assess implant or capsule rupture, but MRI is the most definitive investigative study.

2020

71
Q

A 43-year-old woman presents with moderately large breasts with mild ptosis, and the surgical plan is a bilateral periareolar mastopexy to minimize the length of surgical scars. Which of the following postoperative complications is most commonly associated with this technique?

A) Areolar spreading
B) Constriction of the lower pole of the breast
C) Loss of nipple-areolar sensation
D) Pseudoherniation of the nipple-areolar complex
E) Synmastia

A

The correct response is Option A.

Mastopexy is a procedure designed to improve the appearance of the ptotic breast. The goal is to improve breast shape while minimizing visible scars. The periareolar mastopexy is best suited for correcting very minimal degrees of mammary ptosis. When the procedure is used to attempt to correct moderate to severe ptosis, complications can occur. These include flattening of the central breast mound; widening of the areolar diameter; and irregularity, widening, and even hypertrophy of the circumareolar surgical scar. Additionally, recurrent ptosis, or “bottoming out,” of the breast can occur.

Areolar spreading is the most common complication of this technique. Loss of nipple-areolar sensation is associated with breast tissue resection. Synmastia is associated with large implants, and pseudoherniation of the nipple areolar complex and constriction of the lower pole of the breast are associated with the tuberous breast deformity.

2020

72
Q

A 37-year-old woman with macrotextured saline implants placed 10 years ago was recently diagnosed with a peri-implant fluid collection. Ultrasound-guided aspiration of the fluid is performed and sent for Wright-Giemsa–stained smears, and cell block immunohistochemistry/flow cytometry testing. Images are shown. Which of the following results would confirm a diagnosis of breast-implant–associated anaplastic large cell lymphoma?

A) CD30 negative, ALK negative
B) CD30 negative, ALK positive
C) CD30 positive, ALK negative
D) CD30 positive, ALK positive

A

The correct response is Option C.

This patient has breast implant–associated anaplastic large cell lymphoma (BIA-ALCL) which is a distinct form of CD30-positive T-cell, non-Hodgkin’s lymphoma that arises in association with a breast implant after either reconstructive or cosmetic surgery. The disease is typically contained within the capsule and fluid immediately adjacent to the implant. Patients commonly present with delayed seroma, but can also present with pain, capsular contracture, and/or a palpable mass. BIA-ALCL risk is higher with textured devices and these concerns led to the 2019 FDA recall of Allergan Biocell devices. The recall includes older McGhan and Inamed implants and current Natrelle implants. When patients with suspicious history or symptoms are evaluated, pre-operative imaging (e.g., mammography, ultrasound, and/or MRI) is recommended with aspiration of identifiable fluid or biopsy of mass. Ultrasound is considered the diagnostic modality of choice. Diagnostic evaluation should indicate concern for BIA-ALCL to the pathologist to include cytological evaluation of seroma fluid or mass with Wright-Giemsa stained smears and cell block immunohistochemistry/flow cytometry testing for cluster of differentiation (CD30) and anaplastic lymphoma kinase (ALK) markers. Wright-Giemsa staining shows pleomorphic cells with horseshoe shaped nuclei, nuclear folding and abundant vacuolated cytoplasm. All known cases of BIA-ALCL are CD30-positive and -negative for ALK, distinct from systemic ALCL which is ALK-positive.

2021

73
Q

A 15-year-old girl presents for evaluation of hypoplasia of the right breast. Examination shows an underdeveloped left breast, superiorly displaced nipple-areolar complex, and sunken-appearing chest wall. The pectoralis major muscle and sternum are normal. Which of the following is the most likely diagnosis?

A) Amastia
B) Amazia
C) Anterior thoracic hypoplasia
D) Athelia
E) Poland syndrome

A

The correct response is Option C.

Anterior thoracic hypoplasia is characterized by hypoplasia of the breast in the context of normal sternum and normal pectoralis major muscle. It is on the differential diagnosis of congenital breast deformities, and is distinguished from Poland syndrome by the normal pectoralis major muscle. Pectus excavatum is another condition of the chest wall; it is characterized by abnormal development of the sternum and ribs, and does not affect breast growth, although it can cause medial displacement of the breasts. Pectus excavatum is more common in males than females. Treatment of the breast in Poland syndrome and anterior thoracic hypoplasia depends on the degree of deformity and the goals of the patient; both implant-based reconstruction and fat grafting have been used.

Amazia is an absence of the mammary gland with a present nipple areolar complex, and amastia is complete absence of the mammary gland and nipple areolar complex.

Athelia is an absence of the nipple.

2021

74
Q

A 34-year-old woman desires improvement of the appearance of her breasts and abdomen. Physical examination shows a supernumerary nipple with a small bud of breast tissue just under the fold of the left breast. The embryologic origin and development of this accessory structure occur as a result of which of the following?

A) Incomplete differentiation of the ectodermal ridge
B) Incomplete differentiation of the mesodermal ridge
C) Incomplete involution of the ectodermal ridge
D) Incomplete involution of the mesodermal ridge

A

The correct response is Option C.

In utero, the breasts develop from paired mammary ridges of thickened ectoderm which extend from the axillae to the inguinal regions. The ectoderm will give rise to the nipple and ductal elements, while the mesoderm will eventually give rise to the connective tissue and vascular structures of the breast. Polythelia, or supernumerary nipples, occur as a result of incomplete involution of one of the many epithelial buds along the mammary ridge.

2021

75
Q

A 20-year-old woman presents with concerns about the appearance of her breasts. Examination shows unilateral herniation of the nipple-areola complex and a constricted lower pole. Which of the following maneuvers is most likely to address this patient’s concerns?

A) Elevation of the inframammary fold
B) Radial release of parenchymal bands
C) Skin grafting of nipple-areola complex
D) Vertical mastopexy
E) Wise pattern reduction mammaplasty

A

The correct response is Option B.

The patient description is consistent with tuberous breast deformity. Surgical intervention usually includes reduction of the periareolar herniation with periareolar incisions and radial release of parenchymal bands. Reduction would not address the issues associated with a tuberous breast. Vertical mastopexy alone through standard approaches would potentially further constrict the lower pole. Skin grafting of the nipple-areola complex would not address the tissue herniation. Elevation of the inframammary fold would not address the tuberous deformity and might exacerbate it.

2021

76
Q

An otherwise healthy, 24-year-old woman presents for breast augmentation consultation. Physical examination shows polythelia. On the basis of this finding, which system is most likely to have associated abnormalities?

A) Gastrointestinal
B) Hematological
C) Pulmonary
D) Renal
E) Vertebral

A

The correct response is Option D.

The presence of a supernumerary nipple occurs in 2-6% of females. Polythelia is the presence of two or more supernumerary nipples. A correlation exists between renal disease and polythelia. It is associated with 19% of patients with renal adenocarcinoma and 16.5% of patients with end-stage renal disease. Regular physical examination and urinalysis should be performed in patients with polythelia and any noted abnormality should alert the physician to the need for a renal ultrasound.

2021

77
Q

A 19-year old woman presents with concerns about the appearance of her breasts. Examination shows asymmetrical breasts with grade III ptosis, small breast footprint of both breasts, and a large nipple-areola complex with herniation of breast tissue. Which of the following characteristics of the tuberous breast deformity contributes to the physical findings in this patient?

A) Hyperplasia of one or more quadrants
B) Hyperplasia of only medial quadrants of the breast
C) Hypoplastic areola
D) Low inframammary fold
E) Periareolar ring constriction

A

The correct response is Option E.

Tubular breasts are caused by connective tissue malformations and occur in puberty. Clinical characteristics include breast asymmetry, dense fibrous rings around the areola, hernia bulging of the areola due to hypoplastic fascial support, hypoplasia of one, two or more quadrants, narrowing of the breast base, and a high location of submammary folds.

2021

78
Q

A 32-year-old woman comes to the office 2 years after undergoing bilateral breast augmentation with 350-cc smooth, round saline implants. The patient reports chronic fatigue and joint pain. Physical examination shows soft, symmetrical, and nontender breasts. There are no palpable masses and no axillary adenopathy. Which of the following is most appropriate next step in management?

A) Exchange saline implants for cohesive gel implants
B) Order MRI
C) Perform en bloc removal of the implants
D) Perform stereotactic biopsy of capsule
E) Request autoimmune disease evaluation

A

The correct response is Option E.

Breast implant illness (BII) is a term used to describe a multitude of symptoms seen in patients with breast implants. These symptoms may include the following (as well as others): fatigue, anxiety, headaches, brain fog, anxiety, photosensitivity, hormonal issues, rash, and hair loss. There is no definitive link between these symptoms and breast implants. Research is ongoing.

It is important that all patients with these symptoms are evaluated. BII has been self reported by patients with all types of implant characteristics including silicone, saline, textured surfaces, and smooth-walled implants.

Before undergoing surgery, evaluation of these patients is important. Many of these symptoms can be associated with known autoimmune diseases and should be evaluated for this possibility, either by a rheumatologist or other medical professional. If a known autoimmune disease is diagnosed, then traditional treatment for this known disease should be tried before explantation surgery.

Women who underwent explantation for possible BII had varying degrees of improvement including no improvement, temporary improvement, and permanent resolution of symptoms. In one study, patients with documented autoimmune disease showed no improvement following explantation.

MRI would not initially be needed with a normal physical examination and saline implants.

Stereotactic biopsy would be indicated for palpable masses or breast mass evaluation, not symptoms of BII.

2021

79
Q

A 16-year-old girl is referred to the clinic by her pediatrician for correction of breast asymmetry. The patient reports that her right breast has always been smaller. Physical examination shows mildly shortened right fingers and a Tanner III right breast. Examination shows that the left breast is Tanner IV and no masses. Which of the following is the Mathes and Nahai Classification of the muscle most likely involved in this congenital disorder?

A) Type I
B) Type II
C) Type III
D) Type IV
E) Type V

A

The correct response is Option E.

The pectoralis major has a dual blood supply from both a dominant primary pedicle (pectoral branches of the thoracoacromial artery) and secondary segmental perforators (internal mammary/thoracic perforators).

The absence of the pectoralis major muscle and associated hand deformity is pathognomonic for Poland syndrome. The etiology is unclear but suspected due to vascular interruption during embryogenesis. The diagnosis is often delayed until puberty when asymmetric breast development is noted. Brachydactyly, syndactyly, or ectrodactyly are common and can present with various severity.

Type I flaps have a single dominant blood supply (e.g., rectus femoris or gastrocnemius muscles). Type II flaps have a dominant and minor vascular pedicle (e.g., gracilis or soleus muscles). Type III flaps have 2 dominant pedicles (e.g., pectoralis minor, rectus abdominis and serratus muscles). Type IV flaps only have segmental blood supply (e.g., sartorius or tibialis anterior muscles).

2022

80
Q

During puberty, which of the following hormones is responsible for the development of the glandular buds of the breasts?

A) Estrogen
B) Follicle-stimulating hormone
C) Oxytocin
D) Progesterone
E) Prolactin

A

The correct response is Option D.

During puberty, estrogen controls the growth of the breast ducts. Progesterone controls the growth of the glandular buds. Follicle-stimulating hormone, luteinizing hormone, prolactin, and oxytocin are responsible for milk production.

2022

81
Q

A 22-year-old woman who is dissatisfied with the appearance of her breast seeks surgical correction. Examination shows a constricted base width, widened areola, and areola herniation. Which of the following is the most likely diagnosis?

A) Athelia
B) Hypomastia
C) Poland syndrome
D) Pubertal arrest
E) Tuberous breast

A

The correct response is Option E.

The examination findings are consistent with the diagnosis of tuberous breast. Athelia is defined as absence of the nipple. Hypomastia would include a component of small breast volume, not described in this scenario. Poland syndrome often presents with an underdeveloped chest wall, including potential absence of the pectoralis major muscle. Pubertal arrest might allow for a disproportionately widened areola and apparent herniation if the breast bud had not fully developed, but it would not likely be associated with a constricted breast base width, which demonstrates breast development past the early stages of sexual development (known as Tanner stages).

2022

82
Q

A healthy 34-year-old woman presents to the emergency department for evaluation of a small, open wound of the right breast after an augmentation mastopexy that was performed 4 weeks ago in the Dominican Republic. The wound has not improved with local wound care and empiric treatment with oral antibiotics. She reports no fevers, chills, or malaise. Physical examination shows a 1-cm opening along the vertical incision below the nipple-areola complex with scant serous drainage noted. Which of the following is the most appropriate next step in treatment to address this patient’s findings?

A) Change the local wound care and oral antibiotic regimen
B) Debride the wound and remove the implant in the operating room
C) Excise and close the wound at bedside
D) Order an ultrasound-guided percutaneous drain placement
E) Start a course of intravenous antibiotics

A

The correct response is Option B.

Given this patient’s history of travel to Latin America for surgery, the diagnosis of an atypical mycobacterial infection, such as Mycobacterium abscessus, should be strongly considered. As such, the most appropriate treatment should include operative washout and debridement with removal of the infected prosthesis. Tissue should be sent for acid-fast staining, mycobacterial culture, and pathology.

More conservative treatment of this wound with continued local wound care or attempt at bedside closure will likely delay definitive diagnosis and treatment and lead to treatment failure. Intravenous antibiotics alone are not sufficient to treat an open wound with underlying implant involvement due to an atypical mycobacterial infection. Fluid collection is not suspected so ultrasound-guided drain placement is unnecessary.

2022

83
Q

A 52-year-old woman with a 15-year history of subglandular breast implants presents with concerns about obtaining her first mammogram. She states that she has avoided mammography because she is concerned about implant rupture from the procedure. Which of the following is the most appropriate next step?

A) Diagnostic mammography with Eklund views
B) Diagnostic mammography with Waters views
C) MRI with contrast
D) Screening mammography with Eklund views
E) Screening mammography with Waters views

A

The correct response is Option D.

The correct answer is screening mammography with Eklund views. Some women report that they are hesitant to undergo screening mammography because they are afraid of an implant rupture. A study of breast implant complications reported to the Food and Drug Administration (FDA) demonstrated that of 714 breast implant adverse events reported, 66 described rupture or problems directly associated with mammography. In addition to implant rupture, an oncologic concern is inadequate x-ray views for assessment of the breast tissue.

Eklund described a modified position for mammography in which the breast tissue is displaced in front of the implant. This allows for adequate assessment of the breast tissue. Screening mammography is appropriate for routine mammography; diagnostic mammography is used to further characterize mammographic concerns or in the case of known pathology.

While MRI is used to detect silicone implant rupture and as an adjunct in oncological screening, it is not currently recommended as a routine primary screening tool.

It should be noted that the presence of breast implants should not be used to justify deferral of recommended oncologic screening, and the true incidence of breast implant rupture caused by mammography is difficult to assess given the different generations of implants and the possible presence of capsular contracture.

Waters views are used to assess the maxillary sinuses.

2022

84
Q

An 18-year-old fair-skinned woman presents with a pigmented lesion on her abdominal wall. She reports no symptoms. On examination, a 2-mm lesion appears as a brown-colored protuberance along the milk lines. Which of the following is the embryologic basis for this condition?

A) Anhidrotic ectodermal dysplasia
B) Arrested mammary ridge development
C) Failure of regression of mammary ridges
D) Hypertrophy of glandular tissue
E) Hypoplasia of ectodermal ductal system

A

The correct response is Option C.

Supernumerary nipples (polythelia) occur in 2 to 5% of humans in a position from the groin to the axilla. During the fourth week of embryo development, normally a pair of epidermal thickenings called the mammary ridges develop along the milk lines on either side of the body. These supernumerary nipples can appear similar to pigmented macules or fully developed nipple-areola complexes. These are rarely functioning but can occasionally be a cosmetic issue. Hypertrophy of glandular tissue is macromastia. Arrested mammary ridge development is found during polymastia. Anhidrotic ectodermal dysplasia can be seen in amastia. There is no hypoplasia of ectodermal ductal system in breast development.

2022

85
Q

A 48-year-old woman interested in mastopexy presents with C-cup breasts, grade 3 ptosis, striae of the breast skin bilaterally, 5-mm upper pole pinch thickness, and 7-cm diameter areolae. Which of the following is the strongest CONTRAINDICATION to a circumareolar-only approach to mastopexy?

A) Breast skin striae
B) C-cup breast size
C) Grade 3 ptosis
D) 5-mm upper pole pinch
E) 7-cm diameter areolae

A

The correct response is Option C.

Circumareolar-only approaches to mastopexy are unlikely to be successful with severe ptosis (such as in this patient with grade 3 ptosis) and are usually recommended for patients with only mild-to-moderate ptosis. Moderate breast size (C cup) would not likely impact the success of this surgical approach. The striae are also unlikely to specifically impact a circumareolar approach but may indicate poor skin quality predisposed to recurrence of ptosis corrected with a variety of surgical techniques. The thin upper pole pinch (5 mm) would more likely impact implant-based decision-making. Wide areolae (7 cm) are suitable for circumareolar surgical techniques.

2022

86
Q

A healthy 14-year-old girl presents to the office for evaluation of her breasts. Physical examination shows a superiorly positioned nipple-areola complex with absent breast tissue on the right side and otherwise normal breast development on the left. She wears a B-cup brassiere but uses a brassiere insert on the right. Her mother states that this asymmetry causes the patient significant emotional distress, and they would like to discuss surgical options. Which of the following mammaplasty techniques is most appropriate to address this patient’s right breast findings?

A) Deep inferior epigastric perforator flap
B) Latissimus dorsi myocutaneous flap
C) Saline implant augmentation
D) Structural fat grafting
E) Tissue expander placement

A

The correct response is Option E.

This young, healthy, adolescent girl has amastia of the right breast. Absence of breast tissue along with a superiorly malpositioned nipple-areola complex strongly indicates that tissue expansion may be required prior to formal reconstruction with either an implant or autologous tissue. Since the left breast tissue is still developing, a first-stage right breast tissue expander allows for expansion of the breast pocket, adjustment of breast size as the patient grows, and eventual implant or autologous tissue reconstruction tailored to her body habitus and desires once she reaches maturity.

2022

87
Q

A 31-year-old woman, gravida 2, para 2, presents for augmentation mammaplasty. She wants improved overall fullness with limited scarring. She currently wears a size 34A brassiere. Physical examination shows pseudoptosis. Submuscular augmentation with which of the following techniques is most appropriate to meet this patient’s goals?

A) Pectoralis muscle not released along inframammary fold
B) Release of pectoralis muscle along inframammary fold and periareolar mastopexy
C) Release of pectoralis muscle along inframammary fold and submammary dissection to inferior areola
D) Release of pectoralis muscle along inframammary fold and vertical mastopexy
E) Release of pectoralis muscle along inframammary fold only

A

The correct response is Option C.

The patient would benefit most from a type II dual-plane augmentation mammaplasty. Her breast parenchyma is moderately mobile over the pectoralis muscle, and she has moderate stretch of the lower pole skin. The goal of dual-plane augmentation is to maximize muscle coverage while allowing optimal lower pole expansion. It also allows redistribution of the breast tissue overlying a submuscular implant.

Soft tissue coverage of the implant is an important consideration regarding pocket location. If pinch thickness of the upper breast is 2 cm or greater, an implant can be placed above the pectoralis muscle. If pinch thickness is less than 2 cm, then the implant should be placed at least partially under the pectoralis muscle. If pinch thickness is less than 0.5 cm along the inframammary fold, then the pectoralis muscle should not be released along the inframammary fold.

There are three types of dual-plane augmentation. Type I releases the pectoralis muscle along the inframammary fold. This is used for most routine augmentation mammaplasties, with all the breast parenchyma above the inframammary fold, tight attachments of the parenchyma-muscle interface, and areola-to-inframammary fold stretch of 4 to 6 cm.

Type II dual-plane augmentation releases the pectoralis muscle along the inframammary fold and dissection is performed superficial to the pectoralis muscle to the inferior border of the areola. This is used for augmentation mammaplasties with most of the breast parenchyma above theinframammary fold, looser attachments of the parenchyma-muscle interface, and areola-to-inframammary fold stretch of 5.5 to 6.5 cm.

Type III dual-plane augmentation releases the pectoralis muscle along the inframammary fold, and dissection is performed superficial to the pectoralis muscle to the superior border of the areola. This is used for augmentation mammaplasties in patients with glandular ptosis or true ptosis, when a third of the breast parenchyma is below the inframammary fold, there are very loose attachments of the parenchyma-muscle interface, and areola-to-inframammary fold stretch is 7 to 8 cm. Type III can also be used in breasts with constricted lower poles.

Mastopexy increases scarring, which this patient wanted to limit, and is often not needed with the appropriate dual-plane approach.

2022

88
Q

A 45-year-old woman presents for a discussion regarding breast augmentation. She is concerned about the high incidence of capsular contracture and asks for postoperative antibiotic therapy. Medical history includes no known drug allergies. Which of the following is the most appropriate postoperative antibiotic therapy in this patient?

A) Cephalexin; 500 mg, four times daily for 7 days
B) Ciprofloxacin; 500 mg, twice daily for 7 days
C) Clindamycin; 300 mg, every 6 hours for 7 days
D) Sulfamethoxazole and trimethoprim; one double-strength tablet, twice daily for 7 days
E) Postoperative antibiotics are not indicated

A

The correct response is Option E.

For the patient in the scenario, postoperative (empiric/prophylactic) antibiotics are not indicated. Though capsular contracture is the most common long-term implant complication of breast augmentation, routine postoperative antibiotics have not been shown to decrease this complication. Capsular contracture can result in discomfort, pain, malposition, asymmetry, and the need for revision and reoperation. Though still the subject of hypotheses, the leading theory is that capsular contracture stems from a subacute infection leading to biofilm formation. In one study, the most common isolate was Staphylococcus epidermidis, implying contamination of the implant with insertion. Given this, several algorithms have been proposed to reduce the chance of contamination, including betadine irrigation, antibiotic irrigation, and utilization of a “no-touch.” Postoperative antibiotic therapy has not been demonstrated to be effective in preventing capsular contracture. Thus, all of the answers involving antibiotic prophylaxis are incorrect.

2022